2nd State Exam 24
2nd State Exam 24
2nd State Exam 24
In a patient with infective endocarditis, during treatment with antibiotics, the body temperature has returned to normal, but
the symptoms of severe heart failure are increasing. The patient receives diuretics and cardiac glycosides. Pulse - 112
beats/min. BP140/20 mm Hg. What is your tactic:
perform plasmapheresis
2. After influenza, an 18-year-old boy developed thirst, polyuria, general weakness, blood sugar level 16 mmol/l, 5% in urine,
positive acetone in urine. Type of diabetes the patient has:
3. A 56-year-old man comes to your office because of one-week history of several episodes of left-sided chest pain. The pain is
accompanied by shortness of breath and occurs when he climbs the stairs or walks fast, but it is relieved after about 5 minutes
of rest. He has a history of poorly controlled diabetes, hyperlipidemia, and hypertension. He has smoked one pack of cigarettes
daily for 30 years. His vital signs are within normal limits. Cardiopulmonary examination shows no abnormal findings. An ECG
shows an old left bundle branch block, also present on a previous ECG obtained one year ago. Which of the following is the most
appropriate next step in management?
Reassurance
Analgesics
4. Which of the following is generally NOT recommended in the treatment of acute low back pain?
Spinal manipulation
Physical therapy
Acetaminophen
Bedrest
5. A 47-year-old man presented to his family doctor with the following symptoms: arthritis of the first metatarsophalangeal joint
of the right foot, sharp redness, deformity of this joint, body temperature elevation to 37.6ºC. From the history: a similar attack
of arthritis was noted 3 months ago after consumption of alcoholic beverages, pain worsens at night. What is the most likely
preliminary diagnosis?
Rheumatoid arthritis
Rheumatic arthritis
Primary osteoarthritis
Gouty arthritis
Reactive arthritis
6. A 60-year-old woman complains of pain and stiffness in the right knee joint which is exacerbated by walking or climbing the
stairs and relieved with rest. Which of the following conditions is the most likely diagnosis?
Rheumatoid arthritis
Osteoarthritis
Septic arthritis
Ankylosing spondylitis
Gout
7. A previously healthy 7-year-old boy is brought to the physician because of 2 episodes of vomiting over the past 24 hours. He
had a low-grade fever the day before. He refused to eat breakfast this morning. He is sitting on the office bench and is attentive.
His pulse is 100/min and temperature is 37.1℃. He doesn't have any fevers or rashes. His physical examination shows no
abnormalities. Which of the following is the most appropriate next step in diagnosis?
Uranalysis
Abdominal CT scan
Abdominal X-ray
8. A 12-year-old girl was brought to the doctor due to severe epigastric pain for 2 hours, nausea and vomiting. Her father had a
history of similar episodes of abdominal pain and developed diabetes mellitus at the age of 30 years. Physical examination
revealed peritoneal symptoms. Ultrasound of the abdominal cavity revealed diffuse enlargement of the pancreas; gallstones are
not visualized. Which of the following is the most likely underlying cause of this patient's condition?
is an increase of 15%;
10. A previously healthy 5-year-old boy is brought to the clinic because of fever and dry cough for 2 days. His fever has partially
responded to acetaminophen. He is on her mom's lap playing quietly with a book. His temperature is 38.5℃, respirations are
25/min, and pulse is 110/min. Other than nasal congestion, his physical examination shows no abnormalities. Which of the
following is the most appropriate next step in diagnosis or management of this patient?
Supportive management
Chest X-ray
Empiric antibiotics
Inhaled albuterol
11. A 36-year-old woman who is on chronic pain management with his family physician presents to your office for "lower back
pain." Her pain has gotten worse. It radiates down both her legs, more so on the left. She has had difficulty controlling her bowel
and urine function over the past few days. She has run out of her usual monthly supply of hydrocodone 1 week earlier. Which of
the following symptoms most strongly indicates the need for urgent evaluation?
Patient's age
Urinary incontinence
12. A 72-year-old man with a past medical history of prostate cancer presents with 2 weeks of worsening low back pain and
radiation of pain down his right leg. He does not have any fevers or chills. Upon examination, his straight-leg test is positive at
20 degrees, but his neurological exam is normal otherwise. Which of the following is the strongest indication for spinal imaging
in this patient?
Age > 65
pancreas cancer
chronic colitis
chronic pancreatitis
14. What factors may contribute to the development of thrombosis in the circulatory system?
Hypercoagulation
Hypofibrinogemia
15. A previously healthy 40-year-old gardener presents to your office with 3 days of low back pain, which occurred suddenly
after lifting a bag of compost. The pain has not improved. On physical examination, he has limited range of motion in the lower
back and negative straight leg tests bilaterally. What is the next best step in management?
Bedrest
Short-term analgesics
16. A 40-year-old woman with recurrent pneumonia, productive cough, purulent sputum, and multiple cavitary lesions on chest
x-ray? What is the most likely diagnosis?
Tuberculosis
Bronchiectasis
Lung abscess
Lungs' cancer
17. A 40-year-old patient was admitted to the clinic with complaints of swelling in the legs and arms, weakness, fatigue, and
drowsiness. Sick for a year, to the doctors turned for the first time. Objectively: the patient is pale, lethargic, thyroid gland II
degree, diffuse. The skin of the extremities is dry, cold. Dense edema is palpated in the area of the legs and shoulders, the skin
above them does not gather into a fold, it is pigmented. Respiratory organs without pathology. There is moderate bradycardia
and hypotension. The liver and spleen are not enlarged. Lymph nodes are not palpable. Between what diseases it is necessary
to carry out differential diagnosis?
18. A 16-year-old boy was admitted for massive edema that lasted for 4 months. Pallor, anasarca, pulse 76/min, blood pressure
120/70 mmHg. Art. Blood test showed Hb 130 g/l, platelets 240,000, ESR 24 mm/hour. In urine analysis - rel. density 1024,
protein 16 g/day, leukocytes - 2-4 in the field of view, red blood cells - none, hyaline casts. Total blood protein 43 g/l, albumin 18
g/l, creatinine 120 mmol/l, cholesterol 14.5 mmol/l. What examination method is necessary to establish a diagnosis?
kidney biopsy
chest x-ray
daily proteinuria
19. A 36-year-old woman who is on chronic pain management with her family physician presents to your office for "lower back
pain." Her pain has gotten worse. It radiates down both her legs, more so on the left. She has had difficulty controlling her bowel
and urine function over the past few days. She has run out of her usual monthly supply of hydrocodone 1 week earlier. Which of
the following is the most appropriate next step in management?
Reassurance
20. A 55-year-old man was admitted to the rheumatology department with complaints of pain, limited range of motion in the
knee joints. The pain is mechanical in nature. These complaints have been present for several years with exacerbation over the
last 5 days. Locally: swelling, hyperthermia of the left knee joint, rough crepitus from both sides. Laboratory tests showed:
Hemoglobin – 132 g/L, erythrocytes – 4.7x10^12/L, leukocytes – 6.9x10^9/L, ESR – 25 mm/h. C-reactive protein, rheumatoid
factor – negative. Ultrasound of the knee joint showed signs of arthrosis, bursitis, synovitis on the left. What is the first-line
drug combination suitable in this case?
Glucocorticoids + chondroprotectors
21. A 55-year-old man is found down and unconscious. On physical examination he is afebrile. After catheterization, he passes
a small amount of dark urine. The urine dipstick test for blood is positive but no red blood cells are seen on microscopic
examination of the urine sediment. Which of the following is the most likely diagnosis?
Post-streptococcal glomerulonephritis
Ureteral lithiasis
Rhabdomyolysis
Renal infarction
22. A 63-year-old patient has had pain in the heart area for a year during moderate-intensity physical activity; 2 weeks ago the
pain began to occur with light exertion. Today the pain attack developed at rest and lasted 1.5 hours. It was not treated with
nitroglycerin and was accompanied by cold sweat and weakness. 4 hours after the onset of the attack, he was examined by a
family doctor at home. Make a preliminary diagnosis:
Myocardial infarction
New-onset angina
Cardialgia
23. Patient E, 35 years old, 24 weeks pregnant. No complaints. During the examination, objective data were without any
features. The pulse is rhythmic, 88 per minute, blood pressure is 120- and 80-mm Hg. Art. Heart sounds are rhythmic. The
effleurage symptom is negative on both sides. General blood test: erythrocytes – 3.5*1012, Hb – 118 g/l, leukocytes – 7.2*109,
ESR – 18 mm/h. General urine analysis: showed bacteriuria and leukocyturia. What is the correct tactics for introducing a
pregnant woman?
24. A previously healthy 28-year-old woman presents to your office because of substernal chest pain that comes and goes. It
usually occurs in the evenings after dinner when she lays down to rest. The pain is deep and burning in quality and lasts several
hours. In the mornings, she often has a dry cough and a metallic taste in her mouth but does not particularly feel short of
breath. She has no symptoms during her regular exercise. The patient is not on any medications or smokes. Physical
examination shows no abnormal findings, and an ECG shows no abnormalities. Which of the following is the most appropriate
next step in management?
Antacid medication
Analgesics
Serum D-dimer
Chest X-ray
25. A 32-year-old man was brought to the emergency department of a surgical clinic with complaints of abdominal pain,
uncontrollable vomiting, thirst, severe weakness; he could not even sit. From the anamnesis, it was revealed that over the last 3
months he began to lose weight (only 10 kg) , complained of thirst. Sharp worsening during the last few days. Inspection
data:low nutrition, skin turgor is reduced, the skin and mucous membranes are dry, jams in the corners of the mouth. The
tongue is covered with a brown coating. Heart rate 102 per minute, blood pressure 85/60 mmHg. Art. The abdomen is not
distended upon examination; upon palpation it is painful in all parts with the greatest severity in the epigastric region.
Symptoms of peritoneal irritation are positive. During the examination, vomiting of “coffee grounds” occurred. Specify the
diagnosis with the following examination results: HB 146 g/l, leukocytosis 16 million, glycemia 34 mmol/l, ketone bodies more
than 1000 mg%.
Acute pancreatitis
Gastrointestinal bleeding
26. This is a 41-year-old female with tachypnea. This radiograph demonstrates the typical appearance of which of the following:
No pleural effusion
0,5-1 cm
1-2 cm
2-4 cm
4-6 cm
6-9 cm
28. A 17-year-old patient presented to the clinic with complaints of worsening general condition, frequent episodes of dyspnea,
and fatigue even with mild physical exertion. He has been diagnosed with Chronic Rheumatic Heart Disease (RHD). 1st stage of
aortic valve insufficiency is noted. Laboratory tests revealed the following results: C-reactive protein (CRP) level – 12 mg/L,
rheumatoid factor (RF) – 18 IU/ml. The question arises regarding the dosage of penicillin for this patient. What dose of penicillin
should be administered to this patient?
29. A 30-year-old man was taken to the hospital emergency department with complaints of severe weakness, lethargy, thirst,
pain in the epigastric region, and vomiting. Deterioration of condition after acute respiratory illness. Objectively: lethargic,
lethargic, smell of acetone from the mouth, dry skin, tongue coated with a dirty brown coating. Pulse -100 per minute. Blood
pressure - 100/60 mmHg. Art. Palpation of the abdomen is painful in the epigastric region, there are no symptoms of peritoneal
irritation. Rationale for the diagnosis. What symptom is most characteristic of diabetic syndrome?
Lethargy
Thirst
Epigastric pain
Vomiting
Smell of acetone
30. Which syndrome is characterized by high proteinuria (over 3.5 g / day), hypoproteinemia, hypercholesterolemia, anasarca-
type edema?
Urinary
Dysuric
Acute nephritic
Nephrotic
Hypertensive
Systolic blood pressure ≥ 140 mmHg and/or diastolic blood pressure ≥ 90 mmHg
Systolic blood pressure ≥ 130 mmHg and/or diastolic blood pressure ≥ 80 mmHg
Systolic blood pressure ≥ 135 mmHg and/or diastolic blood pressure ≥ 85 mmHg
Systolic blood pressure ≥ 140 mmHg and/or diastolic blood pressure ≥ 80 mmHg
Systolic blood pressure ≥ 130 mmHg and/or diastolic blood pressure ≥ 70 mmHg
32. A 40-year-old patient came to the clinic with complaints of weakness, fatigue, chilliness, and edema on the face. Sick for a
year after strumectomy. Not treated. Objectively: the patient is lethargic, adynamic, his face is edematous, pale. The tongue is
thick with facets from the teeth. Pulse 50 beats per minute, BP 100/60 mm Hg, skin is dry, cold, scaly, gland tissue is not
detected. What is your preliminary diagnosis?
Postoperative hypothyroidism
Hashimoto disease
Goiter 2 st
Graves disease
Endemic goiter
33. A previously healthy 41-year-old woman presents with 2 weeks of nasal congestion, headaches, general malaise and mild
fevers. Her symptoms of nasal congestion and cough had initially improved after a few days when she started feeling sick again
with worsening nasal congestion, fever and headaches. She denies any shortness of breath. Her vital signs are within the
normal limits. On physical examination, she has mild tenderness over the frontal sinuses. She has mild nasal congestion and
postnasal drip. The remainder of the physical examination is unremarkable. Which of the following is the most likely diagnosis?
Allergic rhinitis
Influenzae
Atypical pneumonia
Rhinosinusitis
Common cold
34. Patient, 35 years old, after suffering from stress, notes increased sweating, irritability, and weight loss. Objectively: height –
170 cm, weight – 55 kg. Hyperhidrosis of the skin and tremor of the fingers of outstretched arms are pronounced. The borders
of the heart are shifted to the left, the sounds are amplified, a systolic murmur is heard at the apex, heart rate is 150 per min,
blood pressure is 140/70 mm Hg. A grade 2 enlargement of the thyroid gland was detected. Fasting glycemia - 6.3 mmol/l.
Blood cholesterol - 7.0 mmol/l. Which of the following research methods is the most informative?
Echocardiography
35. A 44-year-old man complains of weight gain, increased blood pressure, headaches, pain in the spine, and weakness. Ill for
about 2.5 years. Objectively: obesity, wide purplish striae with “minus tissue”, moon face, pseudogynecomastia. MRI: pituitary
adenoma. The concentration of prolactin in the blood is normal, cortisol is increased. What is the most likely diagnosis?
Corticotropinoma
Corticomammatropinoma
Prolactinoma
Incidentaloma
Corticosteroma
36. А 50 year old woman reported with complaints itching on her skin, yellowing of her mucous membranes, pain in her right
hypochondrium, weight loss, and dark blotches on her shoulder and interscapular region. Inspection: yellow color of skin, spider
angioma, clubbing fingers. Jaundice has recently manifested, along with an increase in skin itching. ALT-55 U/l, total bilirubin
52.8 mmol/l, direct bilirubin 24.8 mmol/l are the biochemical analyses.In ultrasound size of portal vein-16mm.Select methods
to determine the degree of liver cirrhosis:
FibroScan
Gastroscopy
Ultrasound
Irrigoscopy
Colonoscopy
37. A 47-year-old man has been complaining for the past few months of compressive pain in the chest that occurs when
walking quickly at an average pace over 500 m, or when climbing stairs to the 2nd floor. Occasionally, pain appears at rest. This
case of angina can be classified as:
unstable angina
progressive angina
variant angina
new-onset angina
38. Which of the following is the most appropriate first-line pharmacotherapy in an adult patient with acute non-radicular neck
pain who has a history of gastric ulcer and no abnormal findings on initial diagnostic evaluation?
Ibuprofen
Acetaminophen
Opiates
Naproxen
Low-dose corticosteroids
39. A 16-year-old man was admitted to the surgical department with acute abdominal pain. Got sick a day ago. The surgeons
suspected an acute abdomen, but noticed the smell of acetone from the mouth. Blood sugar 20 mmol/l. A single portion of
urine contains 8% sugar, acetone +++, specific gravity 1043. The condition is serious. Inhibited. The skin is dry, turgor is
reduced. The tongue is bright and dry. Breathing is noisy. There is hard breathing in the lungs. Pulse 100, rhythmic. Heart
sounds are muffled. BP-90/50 mm Hg. The abdominal wall is tense. Palpation of the abdomen is sharply painful in all parts.
Shchetkin's symptom +. Liver percussion + 6 cm. The patient's body weight is 60 kg. The vomit has a strong smell of acetone.
Which diagnosis assumption would be most correct?
Hypoglycemic coma
Hyperglycemic coma
Diabetes insipidus
Pyelonephritis
Urolithiasis
41. The most likely cause of blindness in a patient suffering from diabetes mellitus for a long time is:
Glaucoma
Cataract
Proliferating retinopathy
Optic atrophy
Autonomic neuropathy
42. A 53-year-old man consulted a doctor about heartburn after a meal, epigastric pains, nausea, sometimes vomiting, acid
belching. Past medical history: has been suffering from this condition for 5 years. He noted increase of these symptoms after
physical effort and forward bending. Objectively: satisfactory condition, clear consciousness. Skin and skin were clean, heart
tones were rhythmic and unchanged. Preliminary diagnosis: gastroesophageal reflux with esophagitis. Which of the following is
the most appropriate first step in the management of this patient?
gastric pH metering
esophageal manometry
esophagogastroscopy
intraesophageal pH-metry
esophagotonokymography
43. A 45-year-old obese woman was accidentally (during a clinical examination) found to have fasting glycemia of 9.2 mmol/l,
glucosuria of 3%, and negative acetone in the urine. The patient's brother suffers from diabetes. Type of diabetes the patient
has:
44. Which of the following blood pressure medications is most likely to cause hypokalemia?
Thiazide diuretics
Beta-blockers
45. A previously healthy 48-year-old woman presents to the clinic with right shoulder pain for the past 2 months. The pain is
worse in the morning, but improves by noon. During this time, she has also had bilateral wrist pain and occasional hip pain. She
is not on any prescribed medications, but takes acetaminophen and ibuprofen in alternating doses with minimal relief. She has a
sedentary lifestyle. Which of the following most strongly indicates serologic testing for rheumatoid arthritis in this patient?
46. These are two chest radiographs taken on the same person 15 minutes apart. What information provided on the images best
explains the apparent improvement in the appearance of the right pleural effusion on the later study?
47. A 68-year-old man from China is brought to the clinic by his daughter for several weeks of persistent cough and occasional
hemoptysis. He has lost significant weight despite following the same diet. He denies shortness of breath, chest pain, or
headaches. He has no history of a serious illness and takes no medications. He has smoked a pack of cigarettes a day for 30
years. Which of the following medical history is most concerning for a serious illness?
Hemoptysis
Findings collection
Smoking history
Persistent cough
Loss of weight
48. A 38-year-old man consulted a doctor with complaints of increased discomfort in the upper abdomen and lack of appetite
for 2 weeks. During this period he had 3-4 episodes of vomiting. Over the past year, there have been frequent attacks of
abdominal pain at night, which decreased after eating. Denies chronic diseases. He smokes one pack of cigarettes a day for 14
years and drinks one to two bottles of beer a day. Medication history: Takes naproxen daily. Temperature 39.5°C, pulse 106
beats per minute, blood pressure 110/70 mmHg. On examination, the abdomen is soft, painful in the right hypochondrium.
Peristaltic sounds are normal. Rectal examination b/o. A stool occult blood test is positive. Hemoglobin level is 128 g/dl, white
blood cell count is 23,100/mm3, platelet count is 230,000/mm3. Abdominal ultrasound: 2 cm hypoechoic mass in the liver.
Which of the following is the most likely cause of these ultrasound findings?
Acute pancreatitis
Echinococcus granulosus
Acute appendicitis
Liver cancer
49. Which of the following types of blood pressure measurements correlates most closely with target-organ damage?
50. A 47-year-old patient was admitted with complaints of interruptions in the heart, pain in the ankle, knee and shoulder joints;
3 weeks before admission there was a fever of 38.5°C, abdominal pain and loose stools for 10 days. Upon admission: ECG
PQ=0.24-0.34 s with loss of QRS, blood leukocytes - 12.9x10^9/l, ESR – 35 mm/h, sialic acid – 270 IU. What disease should you
think about?
cardiac ischemia
rheumatic myocarditis
infectious-allergic myocarditis
dysenteric myocarditis
yersinia myocarditis
51. A 26-year-old woman was accidentally found to have an increase in fasting glycemia to 6.2 mmol/l. Upon repeated
examination, fasting glucose: 5.9 - 6.9 mmol/l. BMI=36kg/m2 What is the most likely diagnosis?
There is no pathology.
52. Which systolic blood pressure value is the target of therapy in most patients with hypertension?
53. A 26-year-old woman comes to the physician because of frequent diarrhea and abdominal cramps over the past six months.
She denies blood or mucous in her stools, changes in her diet or appetite, or weight loss. She has not traveled over the past
year. She has no history of a serious illness and takes no medications. Physical examination shows no abnormalities. Which of
the following is the most likely diagnosis?
Ulcerative colitis
Crohn's disease
Osmotic diarrhea
Secretory diarrhea
54. A 16-year-old boy with increasing difficulty running, jumping, and walking up steps over the past several months states he
was told his grandfather had similar symptoms when he was young. Serum creatine kinase (CK) concentrations are markedly
elevated. Which of the following is the most likely diagnosis?
Poliomyelitis
Dermatomyositis
Churg-Strauss syndrome
55. A 40-year-old patient has a palpable bumpy, dense thyroid gland. According to ultrasound, the volume is 42 ml, the structure
is represented by areas of reduced echogenicity. T4 - 120 nmol/ liter, TSH - 14.9 Antibody titer 283. What is the evidence in
favor of an autoimmune process?
TSH level
T4 level
Antibody titer
Volume on ultrasound
Structure of gland
56. A 52-year-old man presents to your office with 2 weeks of dry, persistent cough. He denies any fevers, shortness of breath,
or sick contacts. He has a history of hypertension. He was started on a new medication 2 months ago when his blood pressure
was elevated. He does not smoke. His vital signs are within normal limits. Cardiopulmonary examination shows no
abnormalities. Which of the following is the most appropriate next step in management?
Chest X-ray
Nasal antihistamine
Review of medications
57. The main indicator according to spirometry to confirm the diagnosis of COPD is:
58. A 45-year-old man presented to the rheumatology department with a history of recurrent episodes of painful arthritis. He
was diagnosed with gout. What dietary restriction is most important for this patient?
59. Which of the following chronic conditions can be exacerbated by a lower respiratory tract infection warranting preemptive
treatment to avoid potential complications?
Asthma
Bronchiectasis
Pulmonary fibrosis
Cystic fibrosis
Chronic bronchitis
60. A 32-year-old woman comes to your office because of fever, dry cough, generalized muscle pain, and headache for 3 days.
She has been unable to focus on work and has had to stay home. Recently, a few of her colleagues have been sick with similar
symptoms. Her temperature is 39℃, pulse is 94/min, and respirations are 15/min. Physical examination shows nasal
congestion and post nasal drip. Her cardiopulmonary examination is otherwise normal. Which of the following history or
physical examination findings is most suggestive of a viral etiology?
Dry cough
Sick contact
Headache
Temperature
61. A 62-year-old man consulted a surgeon at the clinic. He has been suffering from type 2 diabetes mellitus for 12 years;
carbohydrate metabolism was compensated by taking Diabeton. On the 1st toe there is a deep ulcer with an area of necrosis in
the center 2.5 cm in diameter, the bone is swollen and hyperemic. The examination revealed: on both legs, pain, temperature
and tactile sensitivity were slightly reduced. The pulsation on the dorsal artery is practically undetectable. Blood sugar at
treatment was 13.7 mmol/l. General blood test: Hb - 130 g/l, leukocytes - 12x10, ESR 5 Ohm/hour. What complication of
diabetes has developed in the patient?
Diabetic foot
Polyneuropathy
Gangrene
Thrombophlebitis
Phlebothrombosis
62. A 3-year-old boy is having difficulty running, jumping, and walking up steps. Physical examination reveals
pseudohypertrophy of the calf and a waddling gait. Serum creatine kinase (CK) concentrations are markedly elevated. What is
the most likely diagnosis?
Polymyositis
Poliomyelitis
Dermatomyositis
63. A 27-year-old patient was admitted to the ENT department with a diagnosis of tonsillitis. After 3 weeks, weakness, a
decrease in blood pressure to 90/60 mm Hg, pain in the heart area are noted, then paroxysms of supraventricular and
ventricular tachycardia appeared. Cardiomegaly was detected. After 5 weeks, a nasal voice appeared. What disease should you
think about?
infectious-allergic myocarditis
rheumatic myocarditis
dilated cardiomyopathy
exudative pericarditis
64. A patient who suffered an acute myocardial infarction developed chest pain, fever, a pericardial friction rub, an increase in
ESR, and the dynamics of ECG changes without any features. Your conclusion:
idiopathic pericarditis
myocardial rupture
65. A 72-year-old woman was taken to the intensive care unit. Six days ago - a viral infection, after which weakness, thirst,
polyuria, and anorexia appeared and increased. On examination: the condition is very serious, stupor, frequent shallow
breathing, cough. Dry skin and mucous membranes. Blood pressure 130/95 mm Hg. Pulse 140 per minute, atrial fibrillation. No
focal neurological symptoms were identified. Laboratory data: Hb-16 g%, leukocytes 22.0x109, blood potassium 5.5 mmol/l,
creatinine-0.5 mmol/l, blood glucose 65 mmol/l. Glucosuria 2%, acetone (-). How can we explain the normal levels of ketone
bodies and the absence of acetone in the urine with high glycemia?
Concomitant pathology
Age factor
Viral infection
66. A 16-year-old male presents with a history of productive cough and low-grade fever. He was previously healthy and does not
suffer from any chronic ailments. He is diagnosed with tracheobronchitis. What is the next step in management?
Inhaled steroids
Inhaled bronchodilators
67. What signs help to distinguish the nephrotic version of glomerulonephritis from cardiac edema (congestive kidneys)?
enlarged liver
68. A 39-year-old man comes to the physician because of a 6-month history of progressive shortness of breath. He has had a
cough productive of white sputum for 2 years. He smoked 1 pack of cigarettes daily for 16 years but quit 10 years ago. He is in
mild respiratory distress with pursed lips and a barrel chest; he is using the accessory muscles of respiration. Breath sounds
are distant and crackles are present in the lower lung fields bilaterally. Pulmonary function tests show a decreased FEV1: FVC
ratio, increased residual volume, and decreased diffusion capacity. An x-ray of the chest shows hyperinflation and
hypertranslucency of the lower lobes of both lungs. Which of the following is the most likely diagnosis?
Asthma
Bronchiectasis
Cystic fibrosis
Emphysema
69. A 57-year-old Caucasian man presents to your clinic because of an itchy mole on his right shoulder. He is not sure when the
mole appeared, but he noticed it when it started itching 3 months ago. It occasionally bleeds when he scratches it. He thinks it
is darker and bigger than before. On physical examination, the mole is asymmetric with irregular borders and a diameter of 8
mm. Which of the following is the most appropriate next step in diagnosis or management?
Antihistamines
Hydrocortisone cream
Reassurance
70. A 40-year-old patient was admitted with complaints of squeezing pain in the heart area during physical activity, radiating to
the left arm. Duration up to 15 minutes, removable Valocordin (natural sedative drug).The pain has been bothering me for about
8 years. Blood pressure is within normal limits. Examination revealed cardiomegaly and systolic murmur at the apex. With
echocardiography: the thickness of the interventricular septum is 1.5 cm, hypokinesis of the septum, the cavity of the left
ventricle is reduced, the valves are intact. Your suspected diagnosis:
cardiopsychoneurosis
myocarditis
hypertrophic cardiomyopathy
71. This is a 39-year-old female with chronic renal disease. What type of pleural effusion is demonstrated in this frontal and
lateral radiograph?
A laminar effusion
A loculated effusion
A subpulmonic effusion
A hydropneumothorax
Pneumothorax
72. A 73-year-old woman was admitted with a diagnosis of diabetes mellitus and ketoacidotic coma. Consciousness is vague.
According to relatives, he has been suffering from diabetes for 4 years. Previously, I was treated only with diet. For the last two
months I started taking Maninil, 1 tablet. before breakfast and 1 table. before dinner. Treatment with maninil caused increased
appetite. 3 days ago, on the advice of an endocrinologist, the relatives sharply limited the patient’s food intake. By evening, the
patient developed inappropriate behavior and aggressiveness. The next day I had an attack of convulsions, I could no longer
navigate my surroundings and began to refuse food. Hallucinated. This morning I lost consciousness. All these days treatment
with maninil was carried out. Lips are cyanotic. The skin is moist. Tongue is wet. Heart sounds are muffled. Moist fine bubbling
rales are heard in the subscapular region. Atrial fibrillation. Heart rate 120/min. Blood pressure 120/70 mm Hg. Liver +5cm,
swelling of the legs. Increased muscle tone. Clonic and tonic convulsions. Babinski's symptoms ±. Blood sugar 2.0 mmol/l. In
the urine there is acetone ±, sugar is negative. What caused this condition?
Hypoglycemic coma
Ketoacidosis
73. Which of the following heart structures is responsible for generating the electrical impulse that initiates the cardiac cycle?
Sinus node
Aortic valve
Mitral valve
Bundle of His
74. This is a 3-month-old child who has a high fever and cough. Why do you think the right-sided pleural effusion has this
appearance?
75. A 50 year old woman came to the physician with complaints of hypersalivation, acid belching, heartburn, morning
hoarseness of voice, and pain when swallowing. Objectively, the overall state is good. Hight-160cm, weight-88kg. Skin is clean
and moist. The tongue has a yellowish coating on it. The heart beats rhythmically but muffled. Blood pressure was 120/80
mmHg and the pulse rate was 80. Palpation reveals a soft abdomen and minimal tenderness in the epigastric area. a propensity
for constipation. Esophagogastroduodenoscopy: solitary erosions of the distal esophageal mucosa, hyperemic esophageal
mucosa. Which of the subsequent diagnoses is the most probable?
Stomach ulcer
Duodenal ulcer
Esophagitis
Chronic duodenitis
Erosive gastritis
76. A patient complained of an enlarged thyroid gland. Lives in an area where are many cases goiter. I noticed an increase in the
gland 2 years ago, the size does not increase. Objectively: the thyroid gland is enlarged by the eye, the isthmus and lobes are
evenly palpated. Normal skin, no tremor. Pulse 72 per minute, blood pressure 120/70 mm Hg. internal organs without pathology.
The eyes are normal, there are no eye symptoms. What is daily requirement for iodine according to the WHO recommendation?
150-200 mcg
50-70 mcg
400-500 mcg
250-500 mcg
77. A 22-year-old patient, an athlete, was admitted with complaints of fever up to 39°C with chills, shortness of breath with
minor physical exertion, and lack of appetite. I've been sick for about a month. On examination: the skin is icteric, pale,
petechial rashes on the legs. There is a small amount of moist rales in the lungs. Heart sounds are muffled, systolic murmur at
Botkin's point. Heart rate = 106 beats per minute. Blood pressure = 120/40 mm Hg, the liver protrudes from under the edge of
the costal arch by 5 cm, painful on palpation. Minor swelling of the legs. What disease can you think of?
myocarditis
pneumonia
infective endocarditis
rheumatic carditis
78. A 53-year-old man consulted a general practitioner with the following complaints: heartburn after meals, pain in the
epigastrium, nausea, sometimes vomiting, sour belching. Past medical history: has been suffering from this condition for 5
years. He noted increase of these symptoms after physical effort and forward bending. Objectively: satisfactory condition, clear
consciousness. Skin surface was clean, heart tones were rhythmic and unchanged. On esophagogastroduodenoscopy the
patient was diagnosed with gastroesophageal reflux with esophagitis. Which proton pump inhibitor drug is the most effective
according to the latest clinical guidelines for the treatment of gastroesophageal reflux disease?
omeprazole
lansoprazole
pantoprazole
esomeprazole
rabeprazole
79. An 81-year-old man is brought to the physician because of a decrease in appetite and weight loss since his wife died 1 year
ago. He has no other complaints. He has a history of mild hypertension, but takes no medications. Which of the following is the
next best step in the diagnostic evaluation of this patient?
Colonoscopy
Esophagogastroduodenoscopy
80. Select from the following size of portal veins in portal hypertension?
less than 6 mm
6-8 mm
8-10 mm
10-12 mm
13 mm and more
81. Patient A., 24 years old, has been suffering from type 1 diabetes since the age of 7. Over the past 5 years, he has noted a
deterioration in his general condition. General blood test: - erythrocytes – 3.5*1012, Hb-110 g/l, leukocytes – 7.2*109, ESR – 18
mm/h, color. p. – 0.93. General urine analysis - density - 1011, protein - 3.7 g/l, leukocytes - single. Daily protein loss – 7.5 g.
Blood cholesterol 9.1 mmol/l, blood creatinine – 360 µmol/l, fasting capillary blood glucose level 7.8 mmol/l, during the day
within 7.8 – 10, 6 mmol/l. What stage does kidney damage correspond to?
C5
C3
C4
C3 b
C2
82. A 64-year-old woman presented to the emergency department with nausea, vomiting, and substernal chest pain radiating to
the back for 2 hours. Examination of the abdomen reveals tenderness to palpation in the epigastric region. A computed
tomography scan of the patient's chest is indicated. Which of the following diagnoses is most likely?
Esophageal rupture
Aortic dissection
Pneumothorax
83. Which of the following is most likely the main indication for antibiotic treatment in children with group A Streptococcus
pharyngitis?
Reducing symptom severity
84. A 50-year-old man was admitted to the rheumatology department with an acute attack of arthritis of the right ankle joint.
The attack began suddenly in the morning. It is accompanied by a rise in temperature to subfebrile figures. The area around the
joint is swollen, hyperemic, and painful. A month ago, a similar attack was noted with involvement of the first
metatarsophalangeal joint of the right foot after consuming fatty foods and alcohol. A preliminary diagnosis of gout was made.
Presence of monosodium urate crystals in which fluid will confirm the diagnosis of gout?
Synovial fluid;
Urine;
Blood serum;
Cerebrospinal fluid;
Pleural fluid
85. All except which of the following screening tests are generally indicated with a new diagnosis of type 2 diabetes?
Serum creatinine
Echocardiography
Ophthalmologic exam
Lipid profile
86. A 40-year-old patient was admitted to the clinic with complaints of sudden weight loss, irritability, tearfulness, palpitations,
of the heart. 10 years ago she was operated on for diffuse toxic goiter. felt healthy for 9 years. A year ago, after severe
pneumonia, the above complaints appeared. Objectively: the patient is undernourished, the skin is hot and moist. Scar on neck
after strumectomy. The gland tissue is palpated in the isthmus and left lobe. The heart is enlarged to the left, the tones are loud.
Atrial fibrillation. HR=120 per minute Pulse deficit 25 per minute, BP 130/60 mm Hg. the liver is not enlarged, there are no
edema.What is your tactic in treatment?
L-thyroxine
Iodine
Thyreostatics
Glucocorticosteroids
Euthyroxin
87. A 40-year-old patient with angina developed edema, macrohematuria, and increased blood pressure on the 5th day of the
disease. The most likely diagnosis is:
Acute glomerulonephritis
Acute pyelonephritis
Apostematous nephritis
88. Patient B, 22 years old, came to the clinic with complaints of swelling on the face, eyelids, heaviness in the occipital region,
malaise, general weakness, after suffering from a sore throat 2 weeks ago. On examination, the face is pale and puffy, the
eyelids are swollen, and there is mild swelling on the legs and fingers. General urine analysis: hematuria, proteinuria,
leukcitruia. Which drug is the drug of choice in this case?
antibiotics
diuretics
glucocorticoids
cyclophosphamide
89. A patient with congestive heart failure due to dilated cardiomyopathy returned one month after starting therapy with an ACE
inhibitor (enalapril) with complaints of a persistent dry cough. What is the tactic:
90. 70-year-old man with chronic obstructive pulmonary disease (COPD) with exacerbation, respiratory acidosis, shortness of
breath and increased sputum production, treatment options?
Prescribe antibiotics
91. A previously healthy 2-year-old girl was brought to the physician for 1 week of yellowing of the skin, loss of appetite, and 3
episodes of vomiting. Her parents also reported dark urine and brightening fecal masses. For the last 2 days, the girl has been
constantly scratching her stomach and arms, crying a lot. She was born at 38 weeks, pregnancy and childbirth proceeded
without complications. Her family emigrated from Japan 8 years ago. Vaccinations according to the calendar. Vital signs are
within normal limits. On examination, jaundice of the skin and sclera is noted, and a mass is also detected in the right upper
quadrant. Laboratory data: Total bilirubin 5 mg/dL. Direct bilirubin 4.2 mg/dlALT 60 U/l. ACT 40 U/l, GGT110 U/l. Abdominal
ultrasound shows an enlarged gallbladder and fusiform dilatation of the extrahepatic common bile duct. Which of the following
diagnoses is most likely?
Biliary cyst
Mirizzi syndrome
Biliary atresia
Caroli disease
Pancreatic pseudocyst
92. What additional diagnostic test would you order for a 45-year-old woman with sudden onset of pleuritic chest pain?
shortness of breath, hemoptysis, elevated D-dimer levels, recent long-distance travel, and use of oral contraceptives?
Ventilation-perfusion scanning.
Echocardiography
Echocardiogram
93. A patient with hypertrophic cardiomyopathy, diagnosed several years ago, developed syncope. There were no changes in the
ECG compared to the ECG six months ago. It is necessary to assign:
Echocardiography
Coronary angiography
94. A 30-year-old man consulted a general practitioner with complaints of burning pains behind the sternum and heartburn with
no clear connection to physical activity, occurring immediately after a meal and not relieved by nitroglycerin. He has been a
smoker for 6 years and often consumes carbonated beverages. Objectively: Condition was satisfactory. Blood pressure- 120/75
mm Hg. Abdomen was soft, painfulness under the xiphoid process. Esophagogastroscopy was performed and clinical
diagnosis: gastroesophageal reflux disease was established. Which of the following groups of drugs should be preferred first in
the treatment of this patient?
antacids
alginates
prokinetics
antisecretory
acetylcholines
95. Patient B, 22 years old, came to the clinic with complaints of swelling on the face, eyelids, heaviness in the occipital region,
malaise, and general weakness. Increased body temperature to 37.4°. She fell ill 2 weeks after suffering from a sore throat.
General urine analysis: slightly acidic reaction, relative density 1032, protein 3.3 g/l, 47 erythrocytes 15-20 per field of view,
leukocytes up to 8 per field of view, hyaline casts - 6 per field of view, granular casts 3-5 in line of sight. What is the leading
syndrome in the patient?
nephritic syndrome
nephrotic syndrome
poststreptococcal glomerulonephritis
96. Which syndrome dominates and appears in the early stages of biliary cirrhosis?
cholestasis
jaundice
portal hypertension
hypersplenism
hemorrhagic
97. A 35-year-old man is admitted to the rheumatology ward with complaints of weakness in the muscles of the lower limbs.
Four months ago, weakness of the proximal muscles with a sharp increase in muscle enzymes developed. Biopsy revealed
polymyositis, and prednisolone was prescribed at a dose of 60 mg/day. After 6 weeks of treatment, the level of muscle enzymes
returned to normal, and muscle strength appeared. What is the minimal maintenance dose of prednisolone needed to maintain
remission?
5 mg
10 mg
30 mg
40 mg
55 mg
98. A patient with a 2-year history of asthma, previously well controlled, presents for progressive worsening of symptoms in
spite of good adherence to treatment with scheduled steroid inhalers. He has no exposure to pets and is not a smoker.
181.Chest X-ray is unremarkable. What pathology should this patient be worked up for?
Pneumoconiosis
Pulmonary embolism
Bronchiectasis
Tuberculosis
99. A 30-year-old male, non-smoker, with sudden hemoptysis, recent upper respiratory tract infection, and a circular opacity
with cavitation in the right upper lobe on chest x-ray, what is the most likely diagnosis?
Pulmonary embolism
Tuberculosis
lungs' cancer
Aspergilloma
Carcinoma
100. Which of the following does NOT indicate higher severity based on the CURB-65 scoring system for pneumonia?
BP < 60 mm Hg diastolic
Confusion
101. A 65-year-old male comes to the office because of a 1-month history of chest pain and shortness of breath. On
examination, the patient has bilateral crackles. Chest X-ray shows bilateral peripheral lung infiltrates, with normal findings in
central areas of the lungs. His CPR is 230 mg/dL. He has been taking antibiotics for a month but does not respond to treatment.
What is the probable diagnosis?
Acute bronchitis
Influenza
Pulmonary eosinophilia
Lobar pneumonia
102. A 70-year-old man consulted his family doctor complaining of morning stiffness in the knee and wrist joints lasting less
than 30 minutes, pain, and crepitus upon physical exertion. Upon examination, there is no joint deformation, slight tenderness
on palpation of the knee and wrist joints. Laboratory tests revealed an erythrocyte sedimentation rate (ESR) of less than 20
mm/h, and rheumatoid factor was negative. Moderate joint space narrowing and multiple osteophytes were observed on X-ray.
What is the diagnosis of this patient?
Reactive arthritis;
Gout;
Osteoarthritis;
Rheumatoid arthritis
103. A previously healthy 31-year-old woman presents to your office because of stiffness in both her hands, fingers, wrists, and
ankles for several weeks. Her symptoms are worse in the morning but improve significantly by noon. She also complains of dry
eyes and mouth without any changes to her diet or water intake. She feels tired with little daily activity and cannot complete her
work as she used to. She is single and is not in any new relationships. She has a strong family history of rheumatoid arthritis.
Which of the following is the most appropriate next step?
Referring to a psychologist
104. A 36-year-old woman presented with attacks of loss of consciousness, which are preceded by a feeling of hunger, body
trembling, and profuse sweating. Attacks occur more often in the morning, as well as during long breaks in food intake. In
recent weeks they have been accompanied by convulsive syndrome. The examination revealed:fasting glucose 1.2 - 2.0 mmol/l,
during the day - 2.3-3.3 mmol/l. During the attacks, no changes were detected on the ECG, and no focal neurological symptoms
were noted during examination. What diseases of the endocrine system should we differentiate with?
Insulinoma
Ketoacidosis
Hypoglycemic coma
Hypocorticism
105. Patient E, 35 years old, 24 weeks pregnant, went to the clinic for regular screening. No complaints. Objective data without
any features. General urine analysis: slightly acidic reaction, relative density 1032, traces of protein, no red blood cells detected
in the field of view, leukocytes up to 10-15 in the field of view, hyaline casts - 6 in the field of view, granular casts 3-5 in the field
of view, bacteria - +++. What syndrome does a pregnant woman have?
asymptomatic bacteriuria
nephrotic syndrome
nephritic syndrome
poststreptococcal glomerulonephritis
106. A 32-year-old woman comes to your office because of fever, dry cough, generalized muscle pain, and headache for 3 days.
She has been unable to focus on work and has had to stay home. A few of her colleagues have been sick recently. She would
like some medication to help her get better faster. Her temperature is 39℃, pulse is 94/min, and respirations are 15/min.
Physical examination shows nasal congestion and post nasal drip. Her cardiopulmonary examination is otherwise normal.
Which of the following is the most appropriate next step in management?
Empirical antibiotics
Supportive management
Corticosteroids
107. A 46-year-old patient suffering from varicose veins of the lower extremities suddenly developed mixed shortness of
breath, chest pain, and wheezing in the projection of the middle pulmonary field on the right. The ECG records S in I and Q in
standard leads III. Which of the following diseases could cause the above clinical picture?
Spontaneous pneumothorax
Bronchial asthma
Pulmonary embolism
Focal pneumonia
108. 4. Acute renal failure can be caused mainly by antibiotics of the group:
Penicillins
Macrolides
Aminoglycosides
Cephalosporins
Fluoroquinolones
109. A 55-year-old overweight man complains of drowsiness, loud snoring, episodes of apnea, and choking during sleep. What
diagnostic test would you consider for a 50-year-old overweight person?
Polysomnography
Chest X-ray
Electrocardiogram
Angiography
110. Which of the following diagnoses in a patient with low back pain does NOT require urgent care?
Epidural abscess
Vertebral osteomyelitis
Thyrotoxicosis
Hashimoto disease
Autoimmune thyroiditis
Endemic goiter
Hypoparathyroidism
112. A 44-year-old man comes to the office because of indigestion when climbing stairs or carrying heavy packages. The
discomfort happens over his sternum and disappears a few minutes after he stops exertion. He has a 5-year history of
hypertension, abnormal cholesterol values, and a recent upper respiratory infection. He has smoked two packs of cigarettes
daily for 24 years. Which of the following is NOT an indicated therapy for this patient?
Antihypertensive drugs
Cholinergic antagonists
113. 65 years old male, complaints of headaches dizziness. Risk factors: heavy smoker, diabetes, positive family history.
Examination: BP 170/100 mm Hg. What degree of arterial hypertension?
114. Which of the following antidiabetic medications is most likely to cause fluid retention, especially in heart failure?
Thiazolidinediones
Sulfonylureas
Glucagon-like peptide-1 receptor agonists
115. Which of the following is the most appropriate starting dose of basal insulin like NPH, detemir, or glargine?
1 unit/kg/day
3 units/day
3 units/kg/day
1 unit/day
0.3 units/kg/day
116. A 26-year-old primigravida with juvenile myoclonic epilepsy comes to you at 4 months with concern regarding continuing
sodium-valproate treatment. Your advice is:
Switch on to carbamazepine
117. If patient complains for fishy odor and vaginal discharge. What is possible diagnosis?
Gonorrhea
Chlamydiosis
Bacterial vaginosis
Trichomoniasis
Candidiasis
Ovarian cancer
Endometrial cancer
Cervical cancer
Sarcoma of uterus"
119. A 20 years old primigravida comes in emergency at 32 weeks of gestation. She is complaining of blurring of vision, gross
edema. On examination her B.P is 170/115 mm Hg. What is the most likely diagnosis?
Hypertension
Renal disease
Eclampsia
Preeclampsia
Anemia
Reactive
Toxic
Terminal
Vascular
Hyperthermia "
500 mL
1000 mL
1500 mL
50 ml
400 ml
122. A woman in early pregnancy is worried because of some recent discomfort in her left breast. On examination, her skin
appears normal, she has no axillary or clavicular adenopathy, but you palpate a smooth, nontender, 2-cm mass. Your immediate
management should be which of the following?
Breast ultrasound
Mammography
Mammectomy
123. Formula used for estimation of the total iron requirement is:
124. Which of the following histories is not an indication to perform oral glucose tolerance test to diagnose gestational diabetes
mellitus?
Previous Eclampsia
Polyhydramnios
Oligohydramnios
125. A female of 36 weeks gestation presents with hypertension, blurring of vision and headache. Her blood pressure reading
was 180/120 mm Hg and 174/110 mm Hg after 20 minutes. How will you manage the patient?
Admit the patient, start antihypertensives and continue pregnancy till term
Admit the patient, start antihypertensives, MgSO4 and terminate the pregnancy
Caesarian section
Hypertension; Proteinuria
Convulsions
Pain; Hypotension
Hypotension only
127. During first trimester of pregnancy risk of fetal malformation in a pregnant woman with insulin dependent diabetes is best
predicted by?
Level of vitamin D
129. A pregnant female presents with fever. On lab investigation her Hb was decreased (7 mg%), TLC was normal and platelet
count was also decreased. Peripheral smear shows fragmented RBCs. Which is least probable diagnosis?
DIC
TTP
HELLP syndrome
Evans syndrome
Preeclampsia
130. 23 years old primigravida presents to you at fourteen weeks of gestation. She is concerned about normality of fetus. At
what time you will advise her detailed fetal anomaly scan:
14-16 weeks
18-22 weeks
22-24 weeks
24-28 weeks
12 weeks
131. A case of 35 week pregnancy with hydramnios and marked respiratory distress is best treated by:
Intravenous frusemide
Saline infusion
Amniocentesis
ECG of mother
132. A 45-year-old woman is found to have a 4-cm adnexal mass, ascites, and bilateral pleural effusions. Which of the
aforementioned options is the most likely diagnosis?
Leiomyomata
Endometrioma
Follicular cyst
133. Which of the following is an absolute indication for caesarean section in pregnancy associated with heart disease?
Pulmonary stenosis
Coarctation of aorta
Eisenmenger syndrome
Ebstein’s anomaly
Mitral stenosis
134. A 21 year old primigravida is admitted at 39 weeks gestation with painless antepartum hemorrhage. On examination uterus
is soft non-tender and head engaged. The management for her would be:
A speculum examination
Pelvic examination in OT
Antibiotics
135. A pregnant lady acquires chicken pox 3 days prior to delivery. She delivers by normal vaginal route which of the following
statements is true?
136. A 9 month old pregnant lady presents with jaundice and distension, pedal edema after delivering normal baby. Her clinical
condition deteriorates with increasing abdominal distension and severe ascites. Her bilirubin is 5 mg/dl, S. alkaline
phosphatase was 450u/L and ALT (345lu). There is tender hepatomegaly 6cm below costal margin and ascetic fluid show
protein less than 2 mg% Diagnosis is:
HELLP syndrome
Anemia
137. A 27 years primigravida presents with pregnancy induced hypertension with blood pressure of 150/100 mm of Hg at 32
weeks of gestation with no other complications. Subsequently, her blood pressure is controlled on treatment. If there are no
complications, the pregnancy should be terminated at:
40 completed weeks
37 completed weeks
35 completed weeks
34 completed weeks
41 weeks
138. A primigravida at 37 week of gestation reported to labour room with central placenta previa with heavy bleeding per
vaginum. The fetal heart rate was normal at the time of examination. The best management option for her is:
Expectant management
Cesarean section
Vacuum-extraction
139. Which one is the best diagnostic test for cholestasis of pregnancy?
Serum bilirubin
Bile acid
Serum transaminase
Hemoglobin
140. 18 years old P 1 presents in outpatient department ten days after delivery with tender hot painful swelling in right breast.
She also complains of fever with rigors. What will be the most likely management:
Antibiotics.
Analgesics.
Conservative management
Antiallergic tablets
141. A 33-year-old patient has been diagnosed as having adenomyosis. Which of the following symptoms is most consistent
with this diagnosis?
Dyspareunia
Mood swings
Painful defecation
Secondary dysmenorrhea
Pain
25%
50%
75%
100%
10%
143. Which of the following is not a criterion for antenatal diagnosis of Twin-Twin transfusion syndrome?
Dichorionicity
144. Which of the following ovarian tumor is most prone to undergo torsion during pregnancy?
Serous cystadenoma
Mucinous cystadenoma
Dermoid cyst
Follicular cysts
145. A pregnant woman is found to have excessive accumulation of amniotic fluid. Such polyhydramnios is likely to be
associated with all of the following conditions except:
Twinning
Microencephaly
Oesophageal atresia
Anemia
146. At what period does the tuberculosis flare up most commonly in a pregnant patient?
First trimester
Second trimester
Third trimester
Puerperium
Implantation
148. Which vitamin deficiency is most commonly seen in a pregnant mother who is on phenytoin therapy for epilepsy:
Vitamin B6
Vitamin B12
Vitamin A
Folic acid
Vitamin С
149. Which one of the following statements is false regarding “partial hydatidiform mole”?
5.5 - 6.0
6.0 - 6.5
6.5 - 7.0
7.0 - 7.5
1.1 -2.2
151. A woman presents with amenorrhea of 2 months duration; lower abdominal pain, facial pallor, fainting and shock. What is
the most probable diagnosis?
Threatened abortion
Septic abortion
Spontaneous abortion
152. In the evaluation of a 26-year-old patient with 4 months of secondary amenorrhea, you order serum prolactin and β-hCG
assays. The β-hCG test is positive, and the prolactin level is 100 ng/mL (normal is < 25 ng/mL in nonpregnant women in this
assay). This patient requires which of the following?
Computed tomography (CT) scan of her sella turcica to rule out pituitary adenoma
Repeat measurements of serum prolactin to ensure that values do not increase more than 300 ng/mL
153. Infants of diabetic mothers are likely to have the following cardiac anomaly:
Coarctation of aorta
Fallot’s tetralogy
Ebstein’s anomaly
154. A gravida 2 patient with previous LSCS comes at 37 weeks, has BP= 150/100 mm of hg. And on pervaginal examination,
cervix is 50% effaced station-3, os is closed and pelvis is adequate. Proteinuria is +1, Most appropriate step at the moment
would be:
Induce labour
Caesarean sectione
Vacuum extraction
155. Which of the following perinatal infections has the highest risk of fetal infection in the first trimester?
Hepatitis B virus
Syphilis
Toxoplasmosis
Rubella
CMV
Creatinine
Glucose
Lactate
Bilirubin
Albumin
157. A 21-year-old woman has amenorrhea, mild vaginal spotting, pelvic pain, and left shoulder pain. Her vital signs are blood
pressure, 90/50 mm Hg; pulse, 110 bpm; and temperature, 98.6°F. Abdominal examination shows left lower quadrant
tenderness with rebound. Pelvic examination demonstrates a painful 4-cm left adnexal mass. A serum pregnancy test is
positive. A hematocrit is 22% (normal, 35% to 45%). Which of the following is the best next step?
Observation
Progesterone therapy
Methotrexate therapy
Surgery
ECG
3 hours
6 hours
9 hours
12 hours
24 hours
159. 37 years multipara women has a blood picture showing hypochromic anisocytosis. This is most likely indicative of:
Iron deficiency
Hepatitis
160. A 28 years old P2 comes to emergency, after home delivery with heavy bleeding per vaginum. After evaluation and
emergency resuscitation she is diagnosed as a case of uterine atony. What is the appropriate medicine in the management of
this case:
Oxytocin
Salbutamol
Beta blockers
Magnesium sulphate
Nifedipine
161. In which of the following heart diseases maternal mortality is found to be highest?
Eisenmenger’s complex
Coarctation of aorta
Mitral stenosis
Aortic stenosis
Mitral regurgitation
162. A pregnant lady had no complaints but mild cervical lymphadenophathy in first trimester. She was prescribed spiramycin
but she was non-compliant. Baby was born with hydrocephalous and intracerebral calcification. Which of these is likely cause?:
Toxoplasmosis
CMV
Cryptococcus
Rubella
Herpes
163. A 26-year-old pregnant woman has been detected to have a diastolic murmur in mitral area. Echocardiography reveals
mitral valve orifice to be 0.8 cm2. The cause of her murmur is
Functional murmur
164. During examination of patient with PID the cervix was erythematous with characteristic “strawberry” appearance. What is
diagnosis:
Gonorrhea
Chlamidiosis
Siphylis
Trichomoniasis
Candida
165. A 24 years old G 3P2 presents to you at 32 weeks of gestation with preterm prelabour rupture of membranes for ten days.
She is complaining of pain in lower abdomen, fever with rigors and chills and purulent vaginal discharge. What is her diagnosis?
Puerperal pyrexia
Preterm labour
Chorioamnionitis
DUB
166. A patient presents with amenorrhea and galactorrhea. Her PRL levels are elevated. She is not and has never been
pregnant. In addition to evaluating her for a prolactinoma, one also needs to evaluate for other causes that would increase PRL
such as elevated level of which of the following?
Dopamine
Testosterone
Hyperkalemia
Hypercalcemia
Macrocytic anemia
Polycythemia
Tachycardia "
168. A 55-year-old woman presents to your office for consultation regarding her symptoms of menopause. She stopped having
periods 8 months ago and is having severe hot flushes. The hot flushes are causing her considerable stress. What should you
tell her regarding the psychological symptoms of the climacteric?
They are not related to her changing levels of estrogen and progesterone
169. A patient has secondary dysmenorrheal and a fixed pelvis. At laparoscopy, lesions are biopsied that are thought to
represent endometriosis. The diagnosis of endometriosis is confirmed histologically by identifying extragenital implants
containing which of the following?
Fibrosis
170. The dose of anti D gamma globulin given after term delivery for a Rh negative mother and Rh positive baby is:
50 microgram
200 microgram
300 microgram
100 microgram
171. A primipara presents one week after delivery. She is tearful, has spells of cry and lack of appetite and sleep. What is the
most likely diagnosis?
Anaemia
Post-natal depression
Schizophrenia
Maniac disorders
Thalassemia
Uterine sounding,curretage,dilation,anesthesia
Dilation,anesthesia,uterine sounding
173. A 47-year-old woman complains of postcoital bleeding, nearly as heavy as menses. Which of the following is the most
likely origin of her bleeding?
Cervical polyps
Cervical ectropion
Cervical carcinoma
Cervical fibroma
174. A 25-year-old woman is having a severe intrapartum hemorrhage with hypovolemic shock. Which of the following
symptoms is evidence of pituitary infarction?
Infrequent urination
Diarrhea
Easy bruisability
Lactation failure
Constipation
175. A 32 years old G3P2 at 28 weeks of gestation presents with severe pain in the right flank radiating to her groin. She also
complaints of rigors and chills. Urine analysis reveals numerous pus cells. The most likely diagnosis is:
Appendicitis
Pyelonephritis
Meckel’s diverticulum
Anemia
176. Which drug is the best for Gestational trophoblastic disease with jaundice?
Methotrexate
Adriamycin
Actinomycin-D
Cyclophosphamide
Azithromycin
177. A 23 years old primigravida comes in labour room for induction of labour. Cervix is closed and 3 cm long. Which of the
following medicine will be given to her for cervical ripening?
Methergin
Salbutamol
Prostaglandin E 2
Methyldopa
Oxytocin
178. A pregnant lady is diagnosed to be HBs Ag positive. Which of the following is the best way to prevent infection to the child:
179. Which one of the following is an indication for vaginal delivery in twin pregnancy:
180. The separation of normally situated placenta in a case of multiple pregnancy may be due to the following except:
Sudden shrinkage of the uterus following delivery of the first baby "
181. A female having 6 weeks amenorrhea presents with ovarian cyst. The proper management is:
Immediate ovariotomy
Ovariotomy in postpartum
GI blood loss
Hemolytic anaemia
Thalassemia
I trimester
At time of delivery
Perinatal period
After delivery
184. Which one is the screening test for gestational diabetes mellitus that has highest sensitivity?
Glycosylated Hb
Blood fructosamine
Level of glucosuria"
185. A 27-year-old woman presents to your office complaining of mood swings, depression, irritability, and breast pain each
month in the week prior to her menstrual period. She often calls in sick at work because she cannot function when she has the
symptoms. Which of the following medications is the best option for treating the patient’s problem?
Progesterone
A short-acting benzodiazepine
Estrogen
186. A 16-year-old G0P0 patient reports delayed onset of menses, the sudden onset of severe pain, and syncope. A serum
pregnancy test is negative. Her CBC reveals an Hct of 42% and a WBC of 8,000. Which of the following is the most likely
diagnosis?
Ectopic pregnancy
PID
Appendicitis
Diverticulitis
187. Which of the followings is true for an HIV +ve pregnant woman:
CS elective will decrease transmission to baby; Start ART & continue throughout pregnancy ART is safe for gestation
If she hasn’t received prophylaxis, leave her alone for vaginal delivery
188. During the evaluation of secondary amenorrhea in a 24-year-old woman, hyperprolactinemia is diagnosed. Which of the
following conditions could cause increased circulating prolactin concentration and amenorrhea in this patient?
Stress
Primary hyperthyroidism
Anorexia nervosa
Drug induced
189. Situations of Epidemiology Cases: Patient K., aged 40, was admitted to the clinic of infectious diseases with a preliminary
diagnosed with tick-borne viral encephalitis. Sick for 3 days. The onset of the disease is acute: suddenly increased body
temperature up to 40 ° C, there was an intense headache, accompanied by vomiting, myalgia, paresthesia. Epidemiological
history: a month ago, traveled to a territory endemic in tick-borne encephalitis, and consumed raw goat's milk. Vaccination
history -completed an incomplete course of vaccinations against tick-borne encephalitis. Question: What means are used for
emergency prevention of tick-borne encephalitis?
examination and mutual examination for the presence of sucking ticks after visiting the forest
aerosol;
airborne;
transmissible;
food;
contact household.
191. Situations of Epidemiology Cases: In the city of K., which is free from typhoid fever, 6 cases of the disease were registered.
The population of the city of K. is relatively small, the water supply is partially centralized, pumps are used in most parts of the
city. Multi-storey houses are only in the center of the city. The first cases were identified on June 16. In the future, one or two
and less often three patients a day from different families were hospitalized. In the Sh. family, consisting of four people, on
June 16, a girl M. 12 years old and a boy T. 17 years old fell ill. In the family of V., on June 16, the girl T., 6 years old, and on June
18, the girl Yu., 11 years old, fell ill. In R.'s family, a 3-year-old girl G. and a 5-year-old boy S. fell ill on the same day, June 19.
Families live in neighboring one-story houses that lack running water and sewerage. Water is used from the nearest pump,
which is used by residents of five more houses. Families are friendly with each other. On June 1, the birthday of a 6-year-old girl
T. from the family of V was celebrated. In 70% of patients, the disease proceeded in a severe form. Question: What is the route
of infection transmission and your preliminary diagnosis?
Presumably, the epidemic outbreak is due to the airborne route. Epidemic flu outbreak in the city.
Presumably, the epidemic outbreak is due to the food route. Epidemic hepatitis A outbreak in the city.
Presumably, the epidemic outbreak is due to the food route. Epidemic outbreak in the city typhoid fever.
Presumably, the epidemic outbreak is due to the airborne route. Epidemic measles outbreak in the city.
Presumably, the epidemic outbreak is due to the food route. Cholera outbreak in the city.
192. What to do if there are severe reactions in more than 4% of cases with one series of vaccine?
stop using;
continue vaccinations;
193. Situations of Epidemiology Cases: Hepatitis B was diagnosed in a 32-year-old woman who was in a surgical hospital 5
months ago due to a severe injury, where she was repeatedly transfused with blood and blood substitutes. There are 3 more
people in the family: husband is a technician, mother is a pensioner, son, 3 years old, does not attend kindergarten. The patient
and her husband are personnel donors. Task: Make a plan of anti-epidemic measures.
Inspection of medical facilities for compliance with the disinfection and sterilization regime.
Isolation of the patient within the apartment; Room ventilation, UV irradiation, wet cleaning
Examination of the patient, treatment in the infectious department, examination of medical facilities for compliance with the
disinfection and sterilization regime, examination of contacts, dynamic observation for 6 months.
Emergency notification, isolation in an infectious diseases hospital, identification, serological examination and observation of
contacts. Regime-restrictive measures, emergency prevention. San lumen work.
Identification, serological examination and observation of contacts, emergency notification, isolation of patients at home (if it
is possible to comply with the anti-epidemic regimen), or to an infectious diseases hospital. Regime-restrictive measures, em
194. A small purulent discharge from a pustule on the shoulder at the injection site of the BCG vaccine appeared in a newborn.
How to evaluate this phenomenon?
individual intolerance;
genetic disease.
196. . When registering isolated cases of infectious diseases, the territory is assessed as:
prosperous;
emergency;
dysfunctiona;l
sustainable;
not stable.
197. Identify the main age groups at risk for viral hepatitis A (years):
1 – 14;
15 - 30 ;
31 - 45 ;
46 - 65 ;
66- 80.
198. What activities should be carried out in the clinic during the epidemic rise in the incidence of influenza?
Inspection of medical facilities for compliance with the disinfection and sterilization regime.
199. Situations of Epidemiology Cases: A 25-year-old patient was admitted to the clinic of infectious diseases with a
preliminary diagnosed with food poisoning. Clinical data: body temperature - 39.5 ° C, headache, chills, aches in body, nausea,
repeated vomiting, diarrhea. Epidemiological data: the day before the disease, she consumed cakes with cream. Within 2 days,
4 more similar cases were registered in the region. During an epidemiological survey of a cafe where the sick people used
cakes, a panaritium was found on the confectioner's finger. Question: What microorganisms could cause food poisoning in this
situation toxic infection?
Considering panaritium in a confectioner, most likely, infection of a confectionery Staphylococcus aureus products.
Considering panaritium in a confectioner, most likely, infection of a confectionery Escherichia coli products
Considering panaritium in a confectioner, most likely, infection of a confectionery Mycobacterium tuberculosis products.
200. Destruction of probably accumulated microorganisms in the absence of a visible source of infection is disinfection:
preventive:
focal;
current;
final;
Initial.
201. How often you need to change the catheter for the sanitation of the endotracheal tube:
once a day;
change as needed;
can't be changed.
202. What is the correct definition of the term "health care management"?
the science of the ability to use material, financial and human resources for the most efficient operation of the industry;
medical tourism;
204. Situations of Epidemiology Cases: A 25-year-old patient was admitted to the clinic of infectious diseases with a
preliminary diagnosed with food poisoning. Clinical data: body temperature - 39.5 ° C, headache, chills, aches in body, nausea,
repeated vomiting, diarrhea. Epidemiological data: the day before the disease, she consumed cakes with cream. Within 2 days,
4 more similar cases were registered in the region. During an epidemiological survey of a cafe where the sick people used
cakes, a panaritium was found on the confectioner's finger. Question: State a hypothesis about the way of transmission of
pathogens of food poisoning in this situation.
aerogenic
contact
food
blood contact
fecal-oral
205. Situations of Epidemiology Cases: Patient K., aged 40, was admitted to the clinic of infectious diseases with a preliminary
diagnosed with tick-borne viral encephalitis. Sick for 3 days. The onset of the disease is acute: suddenly increased body
temperature up to 40 ° C, there was an intense headache, accompanied by vomiting, myalgia, paresthesia. Epidemiological
history: a month ago, traveled to a territory endemic in tick-borne encephalitis, and consumed raw goat's milk. Vaccination
history -completed an incomplete course of vaccinations against tick-borne encephalitis. Question: Who is the main carrier of
tick-borne encephalitis virus?
dogs ticks
ixodid ticks.
argasid ticks
gamasid ticks
anopheles mosquitoes
digestive system infections that are not present in the patient at the time of admission to the hospital, but develop during his
stay in the hospital.
infections that are present in the patient at the time of admission to the hospital, and develop during their stay in the hospital.
infections that are present in medical personnel at the time of admission to the hospital, and develop during their stay in the
hospital.
infections that are absent in the patient at the time of admission to the hospital, but develop during his stay in the hospital.
respiratory tract infections that are present in patients at the time of admission to the hospital and develop during their stay in
the hospital.
208. Situations of Epidemiology Cases: Child S., 2 years old, attends a kindergarten, nursery group. On October 19, in the
evening, the mother discovered an increase in temperature to 37.5°C and small punctate rash on the child's body. When
contacting a pediatrician On October 20, the doctor made a preliminary diagnosis of measles. Epidemiological data: On October
10, the child with his parents visited cinema. The child's mother had measles in childhood, the father was not sick and was
notvaccinated. No cases of measles have been registered in the kindergarten in the last 2 months. Question: What activities
need to be carried out for contact persons in the children's garden?
Vaccinate all those who have not been vaccinated before this time;
Assign observation of contacts and introduce quarantine for a period of 17 (21) days.
Find out the immune status of children and team members; Vaccinate all those who have not been vaccinated before this time;
Assign observation of contacts and introduce quarantine for a period of 17 (21) days. Find out the immune status of children
and team members;
Find out the immune status of children and team members;Vaccinate all those who have not been vaccinated before this time;
Assign observation of contacts and introduce quarantine for a period of 17 (21) days.
209. Identify who is responsible for compliance with infection control requirements and implementation of measures to prevent
nosocomial infection:
paramedical worker;
medical assistant;
210. What payments do not apply to the payroll of employees of a healthcare organization?
wages of key personnel;
212. What is used to reflect the state of health of the population in official statistics?
case-control studies;
morbidity rate;
survey results;
screening studies;
cohort studies.
213. What family does HIV belong to according to the taxonomic classification of viruses?
adenoviruses;
retroviruses;
rotaviruses;
togoviruses;
arboviruses.
weakened adults;
unvaccinated children;
over-ill adolescents;
infants;
elderly.
215. Situations of Epidemiology Cases: In a children's combine in one of the 7 functioning groups, 5 out of 18 children aged 6-7
years who were in them fell ill. The first 4 children fell ill with AII on the same day directly in the children's institution for a
relatively short period of time from 12.00 to 17.00. The diseases proceeded according to the type of food intoxication
(temperature up to 38-40o, vomiting, loose stools up to 5 times with streaks of blood and mucus in some children). All four
patients were diagnosed with gastroenteritis. In the fifth victim, the disease occurred on the same day as in other children, but
not in a children's institution, but only after returning home (from 19.00) and proceeded much easier (single vomiting, single
liquid stool without impurities, normal temperature). Task: How did the infection spread in the group?
Since in kindergarten children are in close contact with each other, we can say about contact household transmission of
infection
Given the simultaneous onset of diseases, the same type of clinical picture according to the type of food intoxication , we can
say that there is a food way of infection transmission
It is difficult to draw epidemiological conclusions because we do not have enough data on the situation
We can suspect the contamination of the water that the children used
216. Situations of Epidemiology Cases: Patient N., 78 years old, a resident of the city of Saratov, sought medical help 9
November due to fever up to 39 ° C, chills. Primary diagnosis "malaria?". It is known from the anamnesis that patient N. did not
travel to the tropics. October 7th to 9th November was on inpatient treatment in one of the hospitals in the city of Saratov, where
repeatedly received blood transfusions. In the patient's blood rings and gametocytes of Pl. Falciparum. Tropical malaria
diagnosis. Question: How did patient N. contract malaria in this case?
The source of three-day malaria for patient N. was a donor – his a relative of A. who appears to be a carrier of pl.Vivax.
In this case, patient N. was infected with tropical malaria transfusion of infected blood (parenteral route of transmission).
The source of tropical malaria for patient N was a nurse from another patient diagnosed with malaria.
The source of tropical malaria for patient N was relatives who had traveled to tropical countries.
217. According to WHO experts, interventions to combat the increase in cardiovascular morbidity should focus on measures:
primary prevention;
wild carnivores;
domestic pigs;
cattle;
human;
pets.
219. How the destruction of unused opened ampoules containing the remnants of live vaccines is carried out?
only disposal.
public administration;
once;
twice.
223. Explain what to do if, when opening a box with measles vaccine, it did not contain instructions for using the drug?
take advice from another doctor;
224. About the identification of what diseases is reported by the registration statistical form No. 090 / y?
225. Situations of Epidemiology Cases: Child 6 months old, aged 3 and 4.5 months. was vaccinated against polio. The second
vaccination was accompanied by neurological disorders Task: Doctor tactics
226. What does not apply to the ways of protecting the rights of the patient in case of harm to the patient in case of harm to
health?
restoration of the situation that existed before the violation of the right, and suppression of actions that violate the right or
create a threat of its violation;
damages;
227. . Situations of Epidemiology Cases: In a children's combine in one of the 7 functioning groups, 5 out of 18 children aged 6-
7 years who were in them fell ill. The first 4 children fell ill with AII on the same day directly in the children's institution for a
relatively short period of time from 12.00 to 17.00. The diseases proceeded according to the type of food intoxication
(temperature up to 38-40o, vomiting, loose stools up to 5 times with streaks of blood and mucus in some children). All four
patients were diagnosed with gastroenteritis. In the fifth victim, the disease occurred on the same day as in other children, but
not in a children's institution, but only after returning home (from 19.00) and proceeded much easier (single vomiting, single
liquid stool without impurities, normal temperature). Task: How can one explain the occurrence of the fifth disease in a relatively
late period (only after returning home from the kindergarten)?
The occurrence of the fifth disease in a relatively late period (after returning home) is explained that the child has strong
immune system
This indicates an unknown intestinal disease, otherwise all children would get sick at the same time
The occurrence of the fifth disease in a relatively late period (after returning home) is explained by the relatively small dose of
the pathogen received by this child (a very small amount of the infected product was consumed).
The occurrence of the fifth disease in a relatively late period (after returning home) is explained that the child ate other food
228. Which medical instruments are classified as "semi-critical" according to the Spalding classification?
catheters;
bronchoscopes;
implants;
needles;
phonendoscopes.
229. The most “sensitive” indicator of the effectiveness of clinical examination of patients with chronic diseases is:
230. Situations of Epidemiology Cases: Patient K., aged 40, was admitted to the clinic of infectious diseases with a preliminary
diagnosed with tick-borne viral encephalitis. Sick for 3 days. The onset of the disease is acute: suddenly increased body
temperature up to 40 ° C, there was an intense headache, accompanied by vomiting, myalgia, paresthesia. Epidemiological
history: a month ago, traveled to a territory endemic in tick-borne encephalitis, and consumed raw goat's milk. Vaccination
history - completed an incomplete course of vaccinations against tick-borne encephalitis. Question: Ways of transmission of
the causative agent of tick-borne encephalitis sick K.
aerogenic
contact
food
blood contact
fecal-oral
231. Situations of Epidemiology Cases: A 37-year-old patient with chronic hepatitis B has been registered at the dispensary for
2 years (chronic hepatitis B with low replicative activity). He lives in a comfortable apartment with his wife's relatives. His wife
works in the biochemical laboratory of the city clinical hospital (laboratory assistant). Mother-in-law is a general practitioner;
currently working in the clinic of veterans of the Great Patriotic War. Father-in-law is an artist, works in the creative association
"Rainbow". Task: Make a plan of anti-epidemic measures.
Treatment of the patient, compliance with personal hygiene, current disinfection at the place of residence, observation of
contacts, examination.
Identification, serological examination and observation of contacts, emergency notification, isolation of patients at home (if it
is possible to comply with the anti-epidemic regimen), or to an infectious diseases hospital. Regime-restrictive measures, em
Isolation of the patient within the apartment; Room ventilation, UV irradiation, wet cleaning
Emergency notification, isolation in an infectious diseases hospital, identification, serological examination and observation of
contacts. Regime-restrictive measures, emergency prevention. San lumen work.
Emergency notification, serological examination and observation of contacts, prohibit the use of the air conditioning system
232. . What should be done in case of turbidity of the immunoglobulin in the ampoule?
let settle;
can be used;
dispose of;
a science that determines the patterns of occurrence and spread of diseases of various etiologies in order to develop methods
for disease control and preventive measures;
science that determines the patterns of occurrence and spread of infectious diseases;
a science that represents a system of methods for studying the epidemic process in order to establish the causes and
conditions for its development;
a science that determines the patterns of occurrence and spread of non-communicable diseases in order to develop anti-
epidemic and preventive measures;
a science representing a system of methods for studying epidemic diagnostics in order to establish the causes and conditions
for its development.
intensive;
visibility;
ratios;
extensive;
cohort studies.
235. Situations of Epidemiology Cases: Child S., 2 years old, attends a kindergarten, nursery group. On October 19, in the
evening, the mother discovered an increase in temperature to 37.5°C and small punctate rash on the child's body. When
contacting a pediatrician On October 20, the doctor made a preliminary diagnosis of measles. Epidemiological data: On October
10, the child with his parents visited cinema. The child's mother had measles in childhood, the father was not sick and was not
vaccinated. No cases of measles have been registered in the kindergarten in the last 2 months. Question: What measures
regarding contact should be carried out at the place of residence?
Vaccinate mother with live measles vaccine urgently according to epidemiological testimony.
Vaccinate a child with a live measles vaccine urgently according to epidemiological testimony.
Vaccinate all family members with live measles vaccine urgently according to epidemiological testimony.
Vaccinate the father with a live measles vaccine urgently according to the epidemiological testimony.
236. Situations of Epidemiology Cases: Patient N., 78 years old, a resident of the city of Saratov, sought medical help 9
November due to fever up to 39 ° C, chills. Primary diagnosis "malaria?". It is known from the anamnesis that patient N. did not
travel to the tropics. October 7th to 9th November was on inpatient treatment in one of the hospitals in the city of Saratov, where
repeatedly received blood transfusions. In the patient's blood rings and gametocytes of Pl. Falciparum. Tropical malaria
diagnosis. Question: Who or what became a possible source of malaria for this patient N.?
A possible source of malaria for this patient N. was a donor blood who had malaria or was a parasite carrier of Pl. Falciparum
during blood donation.
The source of tropical malaria for patient N was a nurse who donated blood
The source of three-day malaria for patient N was a donor – his a relative of A. who appears to be a carrier of pl.Vivax.
Infection with malaria occurred through household items is possible, especially through personal hygiene items.
237. What does not apply to the initial data for calculating the standard and planned cost of medical care?
the planned volumes of medical care by types of its provision by a medical organization;
contracts for the supply of material resources, containing the conditions for their implementation and payment;
norms of workload, labor costs, calculations of the staffing level, terms of remuneration, determined in accordance with the
current legislation;
useful life (normative service life) of fixed assets and items included in current assets;
medical practice on the use of treatment methods, the usefulness of which has been proven in benign studies;
pathoanatomical conclusions submitted to the court;
239. Situations of Epidemiology Cases: On August 1, he was transferred to the surgical department of the hospital in the city of
N. frompsychiatric hospital citizen K. 63 years old with complaints of acute abdominal pain,of being "stab with a dagger".
Objectively: there is no bloating, there is a sharp tension in the muscles of the abdominal wall ("board-shaped abdomen"),
positive Shchetkin-Blumberg symptom. K. was urgently operated on the day of admission.From the surgical department 3 days
after the operation, a patient with a diagnosis "typhoid fever?" transferred to the infectious diseases hospital, where, on the
basis of clinical and bacteriological data was diagnosed with typhoid fever.Sick 10th of June was involved in the distribution of
food in this ward. During bacteriological examination patients of the ward identified N. - typhoid carrier. Upon admission to the
hospital patient K. was twice examined for an intestinal group of infections with a negative result. According to relatives,
intestinal disorders were not observed. Sick was in the department for treatment for a year. Question: Where did the infection
occur?
In the hospital
At home
In the ward
Impossible o define
240. Situations of Epidemiology Cases: In a children's combine in one of the 7 functioning groups, 5 out of 18 children aged 6-7
years who were in them fell ill. The first 4 children fell ill with AII on the same day directly in the children's institution for a
relatively short period of time from 12.00 to 17.00. The diseases proceeded according to the type of food intoxication
(temperature up to 38-40o, vomiting, loose stools up to 5 times with streaks of blood and mucus in some children). All four
patients were diagnosed with gastroenteritis. In the fifth victim, the disease occurred on the same day as in other children, but
not in a children's institution, but only after returning home (from 19.00) and proceeded much easier (single vomiting, single
liquid stool without impurities, normal temperature). Task: Why did only a small part of the children in the group get sick (5 out
of 18)?
It is obvious that not all children in the group consumed the infected product, but only the sick ones.
There was not enough dose of the toxin in the food to cause poisoning in all the children.
It is possible that other children will show symptoms in a few days as the toxin has a slow effect.
241. Select the official name of the causative agent of a new coronavirus infection:
2019-nCoV;
MERS-CoV;
SARS-CoV;
SARS-CoV-2;
SARS-CoV-20.
242. Vaccination against whooping cough is subject to children under the age of:
1 year;
3 years;
6 years;
9 years;
12 years.
243. Situations of Epidemiology Cases: A patient came to the emergency room of the hospital for medical help. From the
anamnesis it became known that the patient is infected with HIV. Task: Name the elements of special clothing for medical
personnel working in the mode of possible contact with the patient's blood and other biological fluids.
All manipulations in which contact with the patient's biological fluids can occur are carried out in a personal protective
equipment that includes a medical suit, shoes and gloves
All manipulations in which contact with the patient's biological fluids can occur are carried out in a disposable medical gown,
cap, shoes, goggles and gloves
All manipulations in which contact with the patient's biological fluids can occur are carried out in a surgical gown, rubber
gloves, a hat, and removable shoes
There is no need to wear personal protective equipment except for a medical gown, mask and gloves
All manipulations in which contact with the patient's biological fluids can occur are carried out in a surgical gown, rubber
gloves, a hat, and removable shoes; if there is a risk of splashing, use a mask, goggles, a protective screen, oilcloth or rubber
section of epidemiology, representing a system of methods for studying epidemic phenomena and parameters in order to
prevent diseases;
section of epidemiology, representing a system of methods for studying the epidemic process in order to establish the causes
and conditions for its development;
section of epidemiology, aimed at the system of conducting epidemic research in order to establish the causes and conditions
for the development of diseases;
branch of medicine that studies the epidemiology of infectious diseases in order to carry out anti-epidemic measures;
245. When is the epidemiological examination of the focus of infection carried out:
The epidemiological situation in the settlement can be assessed as an outbreak, possibly water given flooding, which could
lead to deterioration water quality.
The epidemiological situation in the settlement can be assessed as outbreak, possibly due to high contagiousness and
penetration through ventilation systems. Therefore, the infection can occur through ventilation systems
The epidemiological situation in the settlement can be assessed as outbreak, possibly due to high contagiousness with ticks.
It is possible that encephalitis ticks contributed to the infection.
The epidemiological situation in the settlement can be assessed as outbreak, possibly due to high contagiousness with
domestic animals.
The epidemiological situation in the settlement can be assessed as an outbreak, possibly a food one, given the flooding, which
could lead to a deterioration water quality.
247. Situations of Epidemiology Cases: In the village of M. in April, as a result of the flood, a partial flooding of the village. A few
days after the flooding of the populated point to the district hospital began to receive patients (12 people) from the flood zone
with complaints of weakness, headache, fever up to 39.4°C, cramping pains in the abdomen. Stool up to 15 times a day, stools
mucus and blood. All patients before the disease consumed raw water from a local well. Was delivered preliminary diagnosis of
acute intestinal infection. Question: What laboratory tests should be carried out to establish the final diagnosis?
stool culture and serological examination with paired sera should be done
248. The results of a retrospective epidemiological analysis are the basis for:
the ability of a leader to influence subordinates, a means of implementing the organization's mission;
the responsibility of the person for the results of activities, the ability of the leader to influence subordinates, the means of
implementing the mission of the organization;
a means of carrying out the mission of the organization;
250. Select the indicator used to study the epidemiological significance of morbidity:
absolute indicator;
intensive indicator;
visibility indicator;
extensive indicator.
By promoting contamination
253. Male, 33 years old, asthenic. Cholecystectomy 2 years ago. After the operation, 6 months later, heartburn appeared,
epigastric pain 1.5 hours after eating with radiation to the back, takes soda. What is the most likely cause of PCES?
gastritis
pancreatitis
254. In a 23-year-old man with an inguinal hernia, after lifting weights, the hernia stopped being reduced, and severe pain
appeared in the area of the hernial protrusion. On examination, the general condition is satisfactory, the hernial protrusion is
sharply painful on palpation and cannot be reduced into the abdominal cavity. What is the most informative diagnostic method
used to make a diagnosis?
Computed tomography
Physical examination
Laparoscopy
255. During an operation for pancreatic necrosis on the 10th day from the onset of the disease, an inspection of the abdominal
cavity revealed that there was no effusion, there were single spots of steatonecrosis on the greater omentum and peritoneum.
There is a scant amount of cloudy effusion in the omental bursa.The pancreas is enlarged in size, with gray-black areas in the
head, body and tail. Parapancreatic tissue with purulent diffuse impregnation. Other parts and organs of the abdominal cavity
and retroperitoneal tissue are not changed. Specify intraoperative diagnosis:
Pancreatogenic abscess.
Pancreatic pseudocyst.
256. An 80-year-old man has been experiencing difficulty defecating, mucus, and traces of dark blood in his stool for the past
six months. 3 days before admission to the clinic, cramping pains appeared in the abdomen, gases stopped passing, and there
was no stool. The general condition of the patient is serious, the tongue is dry and covered with a white coating. Tachycardia up
to 90 per minute, weak pulse, single extrasystoles. BP 110 and60 mm.Hg The abdomen is evenly swollen, intestinal peristalsis
is sometimes visible to the eye; palpation of the abdomen is soft and moderately painful. There are no bowel sounds. There are
no symptoms of peritoneal irritation. Digital examination of the rectum reveals an empty, gaping ampulla. History of myocardial
infarction, hypertension. Which of the additional research methods should be performed to confirm the clinical diagnosis first?
Emergency colonoscopy
Emergency sigmoidoscopy
Irrigoscopy
257. A man was admitted to the hospital with a diagnosis of acute appendicitis. I fell ill 12 hours ago, when the patient
developed pain in the right iliac region, radiating to both the lower back and the groin region. There was nausea, occasional
vomiting, and increased urination. The patient's condition is satisfactory. Pulse 88 beats per minute. The tongue is a bit dry. The
abdomen in the right half is moderately tense and painful only with deep palpation. The “psoas” symptom is positive. The
Shchetkin-Blumberg symptom is negative. Palpation of the right lumbar region is also somewhat painful. Leukocytes in the
blood—13 x 10.9/l. What preliminary diagnosis is most likely?
Acute cholecystitis
Crohn's disease.
258. A 57-year-old man was admitted to the clinic with complaints of cough, sputum, sometimes streaked with blood,
weakness, sweating, and an increase in body temperature in the evenings up to 37.5 degrees. C. History: twice within 3 months
the patient was hospitalized in a therapeutic hospital with a diagnosis of right-sided pneumonia. After a course of antibiotic
therapy, he was discharged home with a significant improvement in his condition. On examination: the condition is moderate.
The right half of the chest lags behind during breathing; percussion on the right side shows dullness of the percussion sound,
absence of vocal tremor, and weakening of respiratory sounds. An X-ray examination reveals an expansion of the root shadow of
the right lung and atelectasis of the upper lobe of the right lung. What preliminary diagnosis is most likely?
Bronchiectasis.
Pneumonia.
Lung cancer.
259. A 33-year-old woman was admitted to the clinic with complaints of pain, swelling in the left mammary gland, increased
body temperature up to 40°C, chills, and headache. From the anamnesis it is known that the patient gave birth 10 days ago. 24
hours ago, severe pain and swelling appeared in the left mammary gland, at the same time the temperature increased to 40°C,
and chills. Objectively: the left mammary gland is enlarged in size, swelling and redness are noted in the upper outer quadrant,
and here, upon palpation, a painful tumor-like formation measuring 5x5 cm, of dense consistency, with softening in the center is
determined. Painful lymph nodes are identified in the axillary region on the left. Blood test: leukocytes 9.5x109/l, shift of the
leukocyte formula to the left, ESR 30 mm/hour. Determine the tactics for introducing the patient.
Conservative treatment
Radical mastectomy
260. A 30-year-old patient with destructive pancreatitis of alimentary etiology, 25 days after hospitalization, developed hectic
temperature, tachycardia, and chills. General blood test: leukocytes - 17.5x10*9/l, p/y - 4%, s/y - 70%, lymphocytes - 22%,
monocytes - 4%. A slightly painful infiltrate is palpated in the epigastrium and left hypochondrium. A preliminary diagnosis was
made: Acute destructive pancreatitis. What complication are we talking about in this clinical situation:
Pancreatic pseudocyst.
Omentobursostomy
262. How long should a surgeon typically perform the surgical hand scrub?
5 seconds
30 seconds
1 minute
5 minutes
15 minutes
263. A 37-year-old woman was admitted to the emergency department with complaints of sharp, cramping pain in the right iliac
region, which appeared 6 hours ago after taking a laxative, and vomiting. Notes a shortening of the intervals between
contractions - the pain becomes constant. On objective examination: PS-106', blood pressure 90/70 mm. Hg Art. The tongue is
dry with a gray coating, the abdomen is swollen, and tympanitis on percussion. On palpation in the right iliac region there is a
painful, dense, mobile, curved, sausage-shaped formation. Perrectum: bloody contents. What type of intestinal obstruction are
we talking about?
Intussusception
Strangulation
Obturation
Knot formation
Looping
264. Which of the following factors increases the risk of surgical site infections?
Smoking
High
Low
Average
Retroduodenal
Intraduodenal
266. A 40-year-old woman was admitted with complaints of pain in both mammary glands, occurring 2-3 days before
menstruation, and discharge from the right mammary gland. Pain in both mammary glands has been bothering me for 5 years. I
didn’t go to the doctors. Over the past 3 months, I noticed discharge from the right breast. Upon examination, the mammary
glands are developed correctly. The skin over them is not changed. On palpation, diffuse thickening and tenderness of both
mammary glands are noted. No nodules were identified. When pressed, a light, transparent, greenish discharge emerges from
the right nipple. What is the most informative basic diagnostic method that will be used to make a diagnosis?
Mammography.
x-ray
3.Ductography
Ultrasound examination
MRI
267. 3 months after resection of 2/3 of the stomach according to Billroth II in the Hofmeister-Finsterer modification for gastric
ulcermancomplained of weakness that occurs 15 minutes after eating, accompanied by a feeling of heat in the upper half of the
body, sudden sweating. After some time, he begins to experience dizziness, tinnitus, rapid heartbeat, trembling of the limbs,
then a feeling of fatigue, drowsiness, loose stools, and polyuria. No abdominal pain. Attacks are more pronounced when eating
foods rich in carbohydrates. After the operation, the patient does not gain weight. Body weight deficiency is 8 kg. What
preliminary diagnosis is most likely?
Dumping syndrome
Diabetes
Malabsorption syndrome
Enteral failure
268. A 53-year-old woman came to the hospital with complaints ofincreased body temperature to 39-40 0C, headache and
severe pain in the thyroid gland. In the general blood test: erythrocyte sedimentation rate -60 mm/h, leukocytes - 19.9x109/l,
lymphocytes -20%, thyroid-stimulating hormone - 4.0 nmol/l and triiodothyronine -3.5.0. pmol/l. Antibodies to thyroglobulin,
antibodies to thyroid peroxidase are not detected. Make a preliminary diagnosis.
lymphomatous thyroiditis
Riedel's thyroiditis
269. Male 62 years old. He was admitted with complaints of pain in the calf muscles of the left leg that occurred when walking
50-100 metersand disappearing after a short rest. I fell ill 2 years ago, when I first began to notice pain in my left shin when
walking for a long time. Subsequently, the number of meters that the patient could walk without rest progressively decreased.
He did not seek medical help. Objectively: the left leg and foot are pale in color, skin temperature is reduced. There is no hair on
the lower leg. Toenails are dull and brittle. Pulsation on the right lower limb is detected at all points, on the left - only on the
femoral artery. Positive symptoms of Oppel, Samuels, Goldflam. The reason for the absence of pulsation in the left popliteal
artery?
270. A 28-year-old woman was admitted with complaints of irritability, increased fatigue, weight loss, palpitations, and
interruptions in cardiac activity. On palpation, the thyroid gland is enlarged due to both lobes and the isthmus, soft-elastic
consistency, painless. Positive eye symptoms: exophthalmos, rare blinking, convergence disorder. Pulse 110 beats. in a minute.
Ultrasound reveals diffuse enlargement of the thyroid gland, volume – 25 ml. Which diagnostic method is the most informative
in making a diagnosis?
Study of the level of thyroid hormones and the level of thyroid-stimulating hormone
Lymphography
271. In a 23-year-old man with an inguinal hernia, after lifting weights, the hernia stopped being reduced, and severe pain
appeared in the area of the hernial protrusion. On examination, the general condition is satisfactory, the hernial protrusion is
sharply painful on palpation and cannot be reduced into the abdominal cavity. What is your treatment strategy?
Emergency surgery
Antibiotic therapy
Administration of antispasmodics
Cleansing enema
272. In a 23-year-old man with an inguinal hernia, after lifting weights, the hernia stopped being reduced, and severe pain
appeared in the area of the hernial protrusion. On examination, the general condition is satisfactory, the hernial protrusion is
sharply painful on palpation and cannot be reduced into the abdominal cavity. What preliminary diagnosis is most likely?
Intestinal colic
Acute appendicitis
Renal colic
Perforated ulcer
273. A 57-year-old woman was admitted to the surgery department with epigastric pain. After collecting an anamnesis, it turned
out that the patient was often bothered by “late” “night” “hunger” pain in the epigastric region, heartburn, and vomiting of acidic
contents. Positive Mendelian symptom. After instrumental diagnostics, a diagnosis was made: Ulcer of the antrum of the
stomach. What operation at the present stage can be considered the most physiological for an ulcer of the antrum of the
stomach?
proximal gastrectomy
274. In the clinic, after instrumental diagnostic methods, a 50-year-old man was diagnosed with lung cancer. What is the
surgeon's further tactics?
Anti-inflammatory therapy
Surgical treatment.
Dynamic observation.
Radiation therapy.
275. During an operation for pancreatic necrosis on the 10th day from the onset of the disease, an examination of the
abdominal cavity revealed that there was no effusion, there were single spots of steatonecrosis on the greater omentum and
peritoneum. There is a scant amount of cloudy effusion in the omental bursa.The pancreas is enlarged in size, with gray-black
areas in the head, body and tail. Parapancreatic tissue with purulent diffuse impregnation. Other parts and organs of the
abdominal cavity and retroperitoneal tissue are not changed. Specify intraoperative diagnosis:
Pancreatogenic abscess.
Pancreatic pseudocyst.
276. Male 62 years old. He was admitted with complaints of pain in the calf muscles of the left leg that occurred when walking
50-100 metersand disappearing after a short rest. I fell ill 2 years ago, when I first began to notice pain in my left shin when
walking for a long time. Subsequently, the number of meters that the patient could walk without rest progressively decreased.
He did not seek medical help. Objectively: the left leg and foot are pale in color, skin temperature is reduced. There is no hair on
the lower leg. Toenails are dull and brittle. Pulsation on the right lower limb is detected at all points, on the left - only on the
femoral artery. Positive symptoms of Oppel, Samuels, Goldflam. What preliminary diagnosis is most likely?
obliterating atherosclerosis
thromboangiitis obliterans
diabetic macroangiopathy
nonspecific aortoarteritis
diabetic microangiopathy
277. A 45-year-old man with an irreducible inguinal hernia has been experiencing pain in the area of the hernial protrusion for 3
days, an increase in the size of the hernial protrusion, redness and thickening of the skin over the protrusion, an increase in
body temperature to 39 degrees with chills. Objectively – the patient’s condition is of moderate severity, the abdomen is
moderately swollen, painless, the hernial protrusion is sharply painful, the skin over it is swollen and hyperemic. What is your
treatment strategy?
Dynamic observation
Emergency surgery
antibacterial therapy
Minor laceration
Deep abrasion
Puncture wound
279. A 50-year-old man complains of pain in the epigastric region, nausea, and occasional vomiting. Gastroduodenoscopy
revealed a penetrating gastric ulcer, high levels of gastrin in the blood, and high acidity of gastric juice. Specify the cause of the
disease:
glucagonoma
gastrinoma
insulinoma
280. An 18-year-old man entered the clinic on a referral from the military registration and enlistment office. The examination
revealed high blood pressure numbers of 200/120 mmHg. The patient notes that he had previously had high blood pressure
values. Complains of recurrent headaches. I had not been examined or treated before. The condition is satisfactory. The
physique is normal. The muscles of the upper half of the body are well developed. Increased pulsation of the vessels of the
upper half of the body. Heart sounds are rhythmic. A systolic murmur is heard over the heart area and from the back between
the shoulder blades. Blood pressure in the arms is 190/110 mmHg. on the legs 105/60 mmHg. Heart rate 84/min. The pulsation
in the arms is clear, in the legs it is weakened. The liver and spleen are not enlarged. There is no swelling. ECG: sinus rhythm,
levogram, signs of left ventricular hypertrophy. On a chest x-ray: usuration of the lower edge of 3-4 pairs of ribs. What
symptoms are most specific for diagnosis?
shortness of breath
headaches
To prevent contamination
To encourage infection
282. A 40-year-old man, shortly after a diagnostic esophagogastroscopy, developed sharp pain in the chest, radiating to the
back and left shoulder, and a single vomiting mixed with scarlet blood. On examination: the patient's condition is of moderate
severity. The abdomen is soft and painless. Temperature - 37.2, leukocytes - 10,000. What preliminary diagnosis is most likely?
Melory-Weiss syndrome.
283. A 40-year-old woman was admitted with complaints of pain in both mammary glands, occurring 2-3 days before
menstruation, and discharge from the right mammary gland. Pain in both mammary glands has been bothering me for 5 years. I
didn’t go to the doctors. Over the past 3 months, I noticed discharge from the right breast. Upon examination, the mammary
glands are developed correctly. The skin over them is not changed. On palpation, diffuse thickening and tenderness of both
mammary glands are noted. No nodules were identified. When pressed, a light, transparent, greenish discharge emerges from
the right nipple. What preliminary diagnosis is most likely?
Diffuse fibrocystic mastopathy
Erysipelas-like cancer
acute mastitis
Nodular mastopathy
284. A 25-year-old woman was admitted with complaints of a tumor-like formation in the left mammary gland, pain and redness
of the skin in the area of the formation, and an increase in body temperature to 38 degrees. C. From the anamnesis it is known
that the patient breastfeeds the child for 6 months after birth. Over the past week, the above complaints have appeared. On
examination, there is hyperemia and skin in the upper outer quadrant of the left mammary gland. Upon palpation over this area,
a tumor-like formation with a diameter of 6-7 cm, densely elastic consistency, with unclear contours, sharply painful on
palpation. The discharge from the nipples is white and odorless. What examination methods are necessary to clarify the
diagnosis?
Mammography
Ultrasound examination
MRI
Chest X-ray
285. A 46-year-old man was admitted with complaints of pain in the calf muscles, buttocks and lower back, which occurred
during the passage of 2030 meters, impotence. Considers himself sick for 9 months. Two goals ago he suffered a small-focal
myocardial infarction. Objectively: the patient’s condition is satisfactory. Vesicular breathing in the lungs. Heart sounds are
muffled. Pulse 72 per minute, correct rhythm. The skin of the lower extremities is ivory-colored. There is no hair on the lower
leg. The pulse in the arteries of the lower extremities is not detected. Positive symptoms of Samuels, Goldflam, Panchenko.
What preliminary diagnosis is most likely?
nonspecific aortoarteritis
thromboangiitis obliterans
Raynaud's disease
286. A 30-year-old woman was admitted with complaints of irritability, increased fatigue, and unmotivated mood swings.
Examination revealed diffuse enlargement of both lobes of the thyroid gland. Positive eye symptoms: widened palpebral
fissures, rare blinking. Pulse - 115 beats/min. For 2 years, she was periodically treated with thyreostatic drugs without much
effect. A diagnosis was made: grade 3 diffuse goiter with symptoms of moderate thyrotoxicosis. Indicate the correct surgical
tactics.
subtotal resection of the thyroid gland after preparation with antithyroid agents;
treatment with radioactive iodine followed by surgery;
288. A 48-year-old woman was admitted with a clinical picture of acute phlegmonous cholecystitis and symptoms of local
peritonitis. The patient was prescribed conservative treatment. Six hours after admission, severe abdominal pain, cold sweat,
and pulse 120 beats appeared. in min.; the abdomen is tense and sharply painful in all parts; the Shchetkin-Blumberg symptom
is positive in all departments. . What complication developed in the patient?
G. acute cholangitis;
289. A 38-year-old man notes that 3 days ago his left limb suddenly became cold, numb, and the pain was increasing. From the
anamnesis: he suffers from rheumatism. Objectively: there is no movement in the toes. Locally, the left lower limb has no
movement in the toes and ankle joint, and there is no deep sensitivity. The shin is increased in volume. The skin is cold, marble-
colored. There is no pulsation at all levels. Determine treatment tactics?
Lumbar sympathectomy
Revascularization osteotrepanation
290. An 18-year-old man entered the clinic on a referral from the military registration and enlistment office. The examination
revealed high blood pressure numbers of 200/120 mmHg. The patient notes that he had previously had high blood pressure
values. Complains of recurrent headaches. I had not been examined or treated before. The condition is satisfactory. The
physique is normal. The muscles of the upper half of the body are well developed. Increased pulsation of the vessels of the
upper half of the body. Heart sounds are rhythmic. A systolic murmur is heard over the heart area and from the back between
the shoulder blades. Blood pressure in the arms is 190/110 mmHg. on the legs 105/60 mmHg. Heart rate 84/min. The pulsation
in the arms is clear, in the legs it is weakened. The liver and spleen are not enlarged. There is no swelling. ECG: sinus rhythm,
levogram, signs of left ventricular hypertrophy. On a chest x-ray: usuration of the lower edge of 3-4 pairs of ribs. What
preliminary diagnosis is most likely?
patent ductus arteriosus
vegetative-vascular dystonia
IHD
291. A 24-year-old man has been experiencing pain in the epigastric region for 3 years, occurring 1-1.5 hours after eating and
at night, and heartburn. Three hours ago, the patient noted the appearance of acute pain in the epigastric region, which spread
throughout the abdomen, and difficulty breathing in an upright position. During hospitalization: the patient's condition is of
moderate severity, he is in a position on the right side with his knees bent; body temperature is 36.8, tachycardia is noted up to
90 per minute. What are the most informative research methods?
colonoscopy;
esophagogastroduodenoscopy;
292. A 48-year-old woman was admitted with a clinical picture of acute phlegmonous cholecystitis and symptoms of local
peritonitis. The patient was prescribed conservative treatment. Six hours after admission, severe abdominal pain, cold sweat,
and pulse 120 beats appeared. in min.; the abdomen is tense and sharply painful in all parts; the Shchetkin-Blumberg symptom
is positive in all departments. The most informative method for diagnosing this disease is:
intravenous cholecystocholangiography;
G. esophagogastroduodenoscopy (EGDS);
laparoscopy.
ice pack;
antibiotics;
antispasmodics;
laxative or enema;
emergency laparotomy.
294. Man. A 49-year-old man was admitted to the surgical department with complaints of abdominal pain and vomiting with a
“fecal” smell. I became acutely ill 3 days ago. The pain was initially cramping in nature, then became constant. There is a history
of surgery for a closed abdominal injury. Objectively: the general condition is severe, pulse 120 per minute. Blood pressure
80/60 mm. Hg Art. Breathing in the lungs is weakened. The tongue is dry like a “brush”, the stomach does not participate in the
act of breathing, it is swollen, tense, and sharply painful. The Shchetkin-Blumberg symptom is positive in all areas. Gas does
not pass, there is no stool. Perrectum: the rectal ampulla is empty, swollen, the sphincter is gaping. What stage of acute
intestinal obstruction are we talking about?
Stages of ischemia;
295. Which of the following jewelry items are acceptable for a surgeon to wear during surgery?
Bracelets
296. A 62-year-old woman was taken for surgery with a diagnosis of pancreatic necrosis and purulent parapancreatitis on the
16th day from the onset of the disease. During inspection of the omental bursa, 200 ml of thick, creamy, odorless pus was
discovered; in the area of the body of the pancreas there are 2 areas 4.0 x 3.0 and 3.0 x 3.0 cm. black in color, clearly
demarcated from healthy pancreatic tissue. Formulate the intraoperative finding:
Phlegmon.
Abscess.
Infiltrate.
Necrosis.
Edema.
297. A 45-year-old man with an irreducible inguinal hernia has been experiencing pain in the area of the hernial protrusion for 3
days, an increase in the size of the hernial protrusion, redness and thickening of the skin over the protrusion, an increase in
body temperature to 39 degrees with chills. Objectively – the patient’s condition is of moderate severity, the abdomen is
moderately swollen, painless, the hernial protrusion is sharply painful, the skin over it is swollen and hyperemic.The most
informative diagnostic method?
Laparoscopy
G. puncture of infiltrate
esophagogastroduodenoscopy.
ECG
300. A man who suffered pancreatic necrosis a month ago developed a tumor-like formation in the epigastrium and left
hypochondrium, dense, slightly painful, measuring 15.0 x 20.0 cm. There was no nausea, vomiting, or weight loss. Body
temperature is 38.7˚ C. X-ray of the stomach shows that the stomach is pushed anteriorly, the duodenal loop is deployed, and
there is a slowdown in the evacuation of barium sulfate from the stomach. Make a preliminary diagnosis:
301. A 40-year-old man was admitted to the surgery department with repeated vomiting mixed with scarlet blood. What
complications can arise if help is not provided in time?
bleeding
mediastinitis.
sepsis
peritonitis
cardiogenic shock.
302. A 68-year-old man with a history of two myocardial infarctions had an attack of sharp pain in the right hypochondrium 3
days ago, which quickly resolved on its own. After 10 hours, pain in the right hypochondrium reappeared, becoming constant
and increasing in nature. There was repeated vomiting of bile. The temperature rose to 38 degrees. Previously, ultrasound
revealed dense inclusions in the gallbladder. The patient's condition is moderate, pulse 98 beats. per minute The tongue is dry
and covered with a white coating. The abdomen is moderately tense and painful in the right hypochondrium, where the bottom
of the gallbladder is palpated. The Ortner and Mussi symptoms are positive, the Shchetkin-Blumberg symptom is positive only
in the right hypochondrium. What is the optimal method for diagnosing this disease?
RPCG;
G. intravenous cholecystocholangiography;
laparoscopy.
303. A 68-year-old man with a history of two myocardial infarctions had an attack of sharp pain in the right hypochondrium 3
days ago, which quickly resolved on its own. After 10 hours, pain in the right hypochondrium reappeared, becoming constant
and increasing in nature. There was repeated vomiting of bile. The temperature rose to 38 degrees. Previously, ultrasound
revealed dense inclusions in the gallbladder. The patient's condition is moderate, pulse 98 beats. per minute The tongue is dry
and covered with a white coating. The abdomen is moderately tense and painful in the right hypochondrium, where the bottom
of the gallbladder is palpated. The Ortner and Mussi symptoms are positive, the Shchetkin-Blumberg symptom is positive only
in the right hypochondrium. The most likely diagnosis for this patient is?
acute cholecysto-pancreatitis;
acute cholangitis.
To reduce sweating
305. What is the primary purpose of elevating a limb with severe bleeding?
To decrease pain
To reduce swelling
To prevent infection
true anthrumectomy
307. A 35-year-old woman suffering from cholelithiasis, after an error in her diet, developed girdle pain in the upper abdomen
and repeated vomiting. Upon examination, the condition is serious. Pulse – 120 beats per minute. Blood pressure – 90/60 mm
Hg. Art. The abdomen is tense and sharply painful in the upper sections; in sloping areas of the abdomen there is dullness of
percussion sound. Intestinal peristalsis cannot be heard. Ultrasound examination: gallbladder without pathologies. Make a
preliminary diagnosis:
308. Why is it important to maintain adequate fluid resuscitation in patients with severe bleeding?
309. A 25-year-old woman was admitted with complaints of a tumor-like formation in the left mammary gland, pain and redness
of the skin in the area of the formation, and an increase in body temperature to 38 degrees. C. From the anamnesis it is known
that the patient breastfeeds the child for 6 months after birth. Over the past week, the above complaints have appeared. On
examination, there is hyperemia and skin in the upper outer quadrant of the left mammary gland. Upon palpation over this area,
a tumor-like formation with a diameter of 6-7 cm, densely elastic consistency, with unclear contours, sharply painful on
palpation. The discharge from the nipples is white and odorless. What preliminary diagnosis is most likely?
Fibroadenoma
Acute mastitis
Mastitis-like cancer
G. Nodular mastopathy
Papilloma
310. A 24-year-old man has been experiencing pain in the epigastric region for 3 years, occurring 1-1.5 hours after eating and
at night, and heartburn. Three hours ago, the patient noted the appearance of acute pain in the epigastric region, which spread
throughout the abdomen, and difficulty breathing in an upright position. During hospitalization: the patient's condition is of
moderate severity, he is in a position on the right side with his knees bent; body temperature is 36.8, tachycardia is noted up to
90 per minute. What is your treatment strategy when confirming the diagnosis?
emergency surgery;
311. A 28-year-old woman was admitted with complaints of irritability, increased fatigue, weight loss, palpitations, and
interruptions in cardiac activity. On palpation, the thyroid gland is enlarged due to both lobes and the isthmus, soft-elastic
consistency, painless. Positive eye symptoms: exophthalmos, rare blinking, convergence disorder. Pulse 110 beats. in a minute.
Ultrasound reveals diffuse enlargement of the thyroid gland, volume – 25 ml. What disease is most likely in this patient?
Acute thyroiditis.
Thyroid cancer
De Quervain's thyroiditis
Thyrotoxic adenoma
312. Which of the following is the most effective method of hand hygiene for a surgeon before performing a surgery?
313. A 24-year-old woman consulted a doctor with signs of thyrotoxicosis; she was worried about irritability, sweating,
weakness, and palpitations. Ill for 2 years. The thyroid gland is enlarged, temperature is 38.8 0C. An X-ray examination of the
chest cavity organs in the anterior mediastinum at the level of the 2nd rib on the right reveals a rounded formation measuring
5x5 cm with clear boundaries. Lung tissue without pathology, in the blood: ESR - 70 mm/h, leukocytes -21x109/l. What disease
can be suspected in the patient?
retrosternal thymoma
314. A 60-year-old woman complains of aching pain in the epigastric region radiating to the back, which has been bothering her
for 4 months and has recently become constant. There is no obvious dyspepsia, but she has lost weight. There is no jaundice or
fever. Palpation tenderness in the epigastric region; no obvious neoplasms are detected. Endoscopy revealed no pathology.
Ultrasound reveals a cystic formation in the projection of the body and tail of the pancreas, with an internal structure with a
diameter of up to60 mm, does not give a Doppler effect, the pancreatic duct is not dilated; There is no pathology of the
hepatobiliary system. What history is significant for making a diagnosis?
weight loss
315. A 55-year-old man was admitted to the surgical department on an emergency basis with massive bleeding from the lower
gastrointestinal tract. Before this episode there were no health complaints. Objectively: the skin and visible mucous membranes
are pale in color. Pulse 78 per minute, blood pressure 110/70 mmHg. The abdomen is soft and painless on palpation. Locally:
During a rectal examination in the knee-elbow position according to the conventional dial, at 7 o’clock there is a node that
bleeds upon contact. At the time of hospitalization there were no signs of ongoing bleeding. In the general blood test:
hemoglobin - 90 g/l, red blood cells 3.5x10¹²/l. Make a preliminary diagnosis?
Rectal cancer
Bleeding hemorrhoids
Rectal polyp
Rectal prolapse
Anal fissure
316. A 42-year-old woman was admitted with complaints of pain along the varicose veins along the medial surface of the upper
third of the right leg, the lower and middle third of the thigh, general malaise, and an increase in temperature to 37.6°C. I've
been sick for two days. The process began after a bruise of the leg and quickly spreads upward. Varicose veins 18 years old. On
examination: pronounced varicose veins of the great saphenous vein system without signs of trophic disorders, hyperemia and
swelling along the vein. Palpation - increased skin temperature and painful thickening along the entire length. A preliminary
diagnosis was made: Varicose veins. Acute thrombophlebitis of the superficial veins. What should the surgeon's tactics be when
admitting such a patient?
translumbar aortoarteriography
Computed tomography
Coronary angiography
Seldinger aortography
Rheovasography
318. What is the primary goal of maintaining a sterile field in healthcare settings?
319. A 34-year-old man was admitted to the clinic with complaints of shortness of breath with slight physical exertion,
interruptions in heart function, and fatigue. It is known from the anamnesis that a heart murmur was detected at birth. Not
previously examined. The condition has worsened over the past 2 years, when shortness of breath during physical activity and
interruptions in heart function began to increase. Objectively: breathing in the lungs is harsh. Heart sounds are rhythmic, single
ventricular extrasystoles. Systolic murmur over the heart region with a maximum in the 2-3 intercostal space to the left of the
sternum, accent of 2 tones above the pulmonary artery. ECG: right gram, signs of right ventricular hypertrophy. Single
ventricular extrasystoles. Right bundle branch block. On echocardiography: the heart valves are unremarkable, there is an
expansion of the right chambers of the heart, turbulent flow at the level of the interatrial septum, with discharge from left to
right. X-ray of the lungs: increased vascular pattern of the lungs, the shadow of the heart is expanded, the 2nd arc is enlarged in
the direct projection. What are the most informative diagnostic methods for determining treatment tactics?
complex echocardiography,
chest x-ray,
ECG
320. What is the correct technique for applying direct pressure to a bleeding wound?
321. A 48-year-old woman consulted a doctor with complaints of a feeling of heaviness, fullness in her legs, and rapid fatigue
during prolonged standing or walking. These phenomena pass rather quickly after the patient assumes a horizontal position.
Has been ill for 11 years. The examination revealed a sharp expansion of the superficial veins of the left leg and thigh with
pronounced pigmentation and trophic disorders of the skin in the distal parts of the limb. Pastosity and slight swelling are also
noted there.Stage the varicose veins of the left lower limb?
322. A 60-year-old woman complains of aching pain in the epigastric region radiating to the back, which has been bothering her
for 4 months and has recently become constant. There is no obvious dyspepsia, but she has lost weight. There is no jaundice or
fever. Palpation tenderness in the epigastric region; no obvious neoplasms are detected. Endoscopy revealed no pathology.
Ultrasound reveals a cystic formation in the projection of the body and tail of the pancreas, with an internal structure with a
diameter of up to60 mm, does not give a Doppler effect, the pancreatic duct is not dilated; There is no pathology of the
hepatobiliary system. Which method will be fundamental for process verification?
B. retrograde cholangiopancreaticography
D. CT or MRI
323. After completing the surgical hand scrub, what is the appropriate method for a surgeon to dry their hands?
324. A 45-year-old man with an irreducible inguinal hernia has been experiencing pain in the area of the hernial protrusion for 3
days, an increase in the size of the hernial protrusion, redness and thickening of the skin over the protrusion, an increase in
body temperature to 39 degrees with chills. Objectively – the patient’s condition is of moderate severity, the abdomen is
moderately swollen, painless, the hernial protrusion is sharply painful, the skin over it is swollen and hyperemic. What
preliminary diagnosis is most likely?
Peritonitis
inguinal lymphadenitis
Acute appendicitis
sliding hernia
325. A man, 18 years old, complains of intense, sharp pain in the perineal area, aggravated by straining, defecation, an increase
in body temperature to 39.0, chills. When examining the anal area, there is hyperemia of the skin to the right of the anus,
swelling and bulging of tissues, and upon palpation there is sharp pain and fluctuation. A diagnosis of acute subcutaneous
paroproctitis is made.What diseases need a differential diagnosis?
acute lymphadenitis
To spread pathogens
To increase contamination
327. What is the correct procedure for applying a tourniquet to control bleeding?
Use a tourniquet only as a last resort when all other methods fail
328. A 43-year-old man was admitted with complaints of cramping abdominal pain, repeated vomiting, retention of stool and
gas. These complaints appeared 6 hours before admission after eating. Objectively: general condition is moderate, pulse 100
per minute, blood pressure 80/60 mm. Hg Art. The abdomen is swollen, painful on palpation. There is a sub-scar from the
xiphoid process to the navel. Peristalsis is enhanced. Perrectum: The ampulla of the rectum is filled with feces. History:
operated on 1 year ago for a perforated duodenal ulcer and peritonitis. What is the reason for this obstruction?
Adhesive disease
Tumors
Foreign bodies
Developmental anomalies
329. A 20-year-old woman mistakenly took a caustic soda solution about 3 months ago. Currently, rapidly progressing
dysphagia has developed. X-ray contrast esophagography with an aqueous suspension of barium sulfate reveals a cicatricial
stricture of the middle third of the esophagus. At the site of narrowing, the diameter of the esophagus does not exceed 2-3 mm.
What treatment method is indicated for the patient?
parenteral nutrition
gastrostomy placement
330. Incomingmanwith atherosclerotic gangrene of the foot with swelling spreading to the lower leg up to the knee joint.
Gangrene is wet in nature and is accompanied by severe endotoxicosis. The patient has several severe concomitant diseases of
the cardiovascular and respiratory systems, and has a history of 2 myocardial infarctions. Moderate diabetes mellitus. Indicate
the doctor's tactics in this clinical case.
Urgent amputation of a limb at the level of the middle or lower third of the thigh is indicated after minimal therapeutic
preparation
Conservative treatment, control of blood sugar levels, correction of concomitant diseases are indicated.
Urgent amputation of a limb at the level of the lower third of the leg after minimal therapeutic preparation is indicated
Urgent amputation of the limb at the level of the upper third of the leg after minimal therapeutic preparation is indicated
331. An 18-year-old man entered the clinic on a referral from the military registration and enlistment office. The examination
revealed high blood pressure numbers of 200/120 mmHg. The patient notes that he had previously had high blood pressure
values. Complains of recurrent headaches. I had not been examined or treated before. The condition is satisfactory. The
physique is normal. The muscles of the upper half of the body are well developed. Increased pulsation of the vessels of the
upper half of the body. Heart sounds are rhythmic. A systolic murmur is heard over the heart area and from the back between
the shoulder blades. Blood pressure in the arms is 190/110 mmHg. on the legs 105/60 mmHg. Heart rate 84/min. The pulsation
in the arms is clear, in the legs it is weakened. The liver and spleen are not enlarged. There is no swelling. ECG: sinus rhythm,
levogram, signs of left ventricular hypertrophy. On a chest x-ray: usuration of the lower edge of 3-4 pairs of ribs. What is the
most informative diagnostic method for making a diagnosis?
chest x-ray
coronary angiography
thoracoscopy
G.+ aortography
lung scintigraphy
332. A 34-year-old man was admitted to the clinic with complaints of shortness of breath with slight physical exertion,
interruptions in heart function, and fatigue. It is known from the anamnesis that a heart murmur was detected at birth. Not
previously examined. The condition has worsened over the past 2 years, when shortness of breath during physical activity and
interruptions in heart function began to increase. Objectively: breathing in the lungs is harsh. Heart sounds are rhythmic, single
ventricular extrasystoles. Systolic murmur over the heart region with a maximum in the 2-3 intercostal space to the left of the
sternum, accent of 2 tones above the pulmonary artery. ECG: right gram, signs of right ventricular hypertrophy. Single
ventricular extrasystoles. Right bundle branch block. On echocardiography: the heart valves are normal, there is an expansion of
the right chambers of the heart, turbulent flow at the level of the interatrial septum, with discharge from left to right. X-ray of the
lungs: increased vascular pattern of the lungs, the shadow of the heart is expanded, the 2nd arc is enlarged in the direct
projection. What preliminary diagnosis is most likely?
pericarditis
333. A 23-year-old woman was admitted to the clinic with complaints of pain, swelling in the left mammary gland, increased
body temperature up to 40°C, chills, and headache. From the anamnesis it is known that the patient gave birth 20 days ago. 28
hours ago, severe pain and swelling appeared in the left mammary gland, at the same time the temperature increased to 40°C,
and chills. Objectively: the left mammary gland is enlarged in size, swelling and redness are noted in the upper outer quadrant,
and here, upon palpation, a painful tumor-like formation measuring 5x5 cm, of dense consistency, with softening in the center is
determined. Painful lymph nodes are identified in the axillary region on the left. Blood test: blood leukocytes 9.5x109/l, ESR 30
mm/hour. What is your suspected diagnosis?
334. What is the term used to describe infections that occur after surgery but are not related to the surgical site?
Superficial infections
Deep infections
Nosocomial infections
Systemic infections
Opportunistic infections
335. A 39-year-old man underwent cholecystectomy 2 days ago for acute phlegmonous cholecystitis. Complains of bloating
and abdominal pain. Objectively: general condition is moderate, pulse 100, blood pressure 90/70 mm. Hg Art. The abdomen is
distended, painful on palpation, peritoneal symptoms are negative. Tympanitis is determined by percussion. There is no
peristalsis. Gas does not pass, there is no stool. What type of intestinal obstruction is involved in this problem?
Dynamic
Mechanical
Spastic
Strangulation
Paralytic
336. A 72-year-old man was admitted to the clinic with complaints of sharp cramping abdominal pain, nausea, vomiting, and
failure to pass gas and feces. The condition constantly worsened: general weakness increased, appetite disappeared, then he
began to notice rumbling in the stomach, periodically difficult passage of gases and feces. A day ago, cramping pain appeared,
gas and feces stopped passing. There was 2 profuse vomiting of gastric and intestinal contents. Previously, I went to the
hospital with fecal blockages, which were removed. In the lungs, breathing is vesicular, there is no wheezing. Percussion -
tympanitis, sluggish peristalsis, negative symptoms of peritoneal irritation. What type of intestinal obstruction?
Dynamic
Mechanical
Spastic
Strangulation
Obstructive
337. A 51-year-old man was diagnosed 5 years ago with a nodular euthyroid goiter measuring 1.3x2 cm in the left lobe of the
thyroid gland. Throughout the years, education did not bother him. Over the past 3 months, education has doubled. The patient
notes a deterioration in his general condition: weakness, fatigue. Laboratory data: leukocytes – 14.6x109/l, erythrocyte
sedimentation rate – 60 mm/h. Antibodies to thyroglobulin -55 IU/ml, antibodies to thyroid peroxidase -45 IU/ml. Thyriotropic
hormone -3.0 mmol/l, total triiodothyronine -4.0 nmol/l. Make a preliminary diagnosis.
Hashimoto's thyroiditis.
Riedel's thyroiditis
thyroid tuberculosis
338. What is the recommended method for controlling bleeding from a scalp laceration?
Encouraging contamination
Preventing contamination
341. Male 62 years old. He was admitted with complaints of pain in the calf muscles of the left leg that occurred when walking
50-100 metersand disappearing after a short rest. I fell ill 2 years ago, when I first began to notice pain in my left shin when
walking for a long time. Subsequently, the number of meters that the patient could walk without rest progressively decreased.
He did not seek medical help. Objectively: the left leg and foot are pale in color, skin temperature is reduced. There is no hair on
the lower leg. Toenails are dull and brittle. Pulsation on the right lower limb is detected at all points, on the left - only on the
femoral artery. Positive symptoms of Oppel, Samuels, Goldflam. What are the possible treatment tactics for this patient:
femoropopliteal bypass
lumbar sympathectomy
femoral-tibial bypass
conservative therapy to improve microcirculation and metabolic processes in tissues and stimulation of collateral
development
343. An 18-year-old man was admitted to the clinic with cramping abdominal pain and repeated vomiting. I fell ill 40 hours ago,
when acute cramping pain in the abdomen and vomiting appeared. A local physician was called, who diagnosed acute gastritis
and prescribed treatment. The next day, the patient continued to have cramping abdominal pain and vomiting. According to the
patient, the day before there was not much stool. The abdomen is moderately swollen, soft on palpation, painful on the right.
Symptoms of Sklyarov-Mathieu, Shchetkin-Blumberg are pronounced. Plain fluoroscopy of the abdominal cavity reveals a large
number of Kloiber cups. Preliminary diagnosis: acute intestinal obstruction. Indicate the treatment tactics in this problem.
Emergency surgery
Planned surgery
Waiting tactics
Conservative treatment
Residential surveillance
344. A 35-year-old woman came to the clinic with complaints of pain, swelling in the right mammary gland, increased body
temperature to 39°C, and general weakness. From the anamnesis it is known that the patient is nursing, the child is 3 months
old. 10 hours ago before admission, the body temperature suddenly increased to 39°C, severe pain appeared in the right
mammary gland, and general weakness. Objectively: the right mammary gland is enlarged in size, a painful infiltrate with
unclear boundaries is detected in its lower-inner quadrant, the skin over it is not changed. A diagnosis of acute lactation
mastitis on the right side was made. Determine the surgeon's tactics.
Conservative treatment
Radical mastectomy
345. A man, 43 years old, is admitted to the clinic. Complaints upon admission: pain in the left chest, shortness of breath,
periodic cough, increased body temperature, general severe weakness. From the anamnesis, he has been sick for a week. He
independently went to the clinic at his place of residence. General blood test - leukocytes - 15 thousand. X-ray of the chest
organs reveals a formation in the left lung measuring 7x5 cm with unclear contours. Consulted with a phthisiatrician -
tuberculosis is excluded. Katsoni's test is positive. What is the complication of pulmonary echinococcosis in this situation?
346. A 36-year-old man, a native of the Naryn region. Complains of shortness of breath during physical activity, pain, heaviness
in the right half of the chest, periodic chills. Considers himself sick for 5 years. He did not seek medical help. Objectively: the
condition is of moderate severity. Nutrition reduced. There is smoothing of the intercostal spaces, dullness of percussion sound
and absence of breathing on the right from the III to V ribs. On a plain X-ray of the chest, in the upper lobe of the right lung there
is an intense rounded shadow 12 by 12 cm with clear, even contours. The leukoformula shows 7% eosinophilia. A preliminary
diagnosis was made: Echinococcosis of the right lung. An additional and definitive method for diagnosing echinococcosis is?
Mantoux test
Katsoni sample
Rehberg's test
Valsalva maneuver
Moshkovich's test
347. A 48-year-old woman was operated on at the clinic for calculous cholecystitis as planned. The postoperative course was
uneventful; after 9 days, she was discharged for outpatient treatment in satisfactory condition. 2 months after the operation, the
patient notes an attack of pain in the right hypochondrium with a rise in temperature to 39 degrees with chills. A day after the
attack, jaundice appeared. The patient was hospitalized in the surgical department. What preliminary diagnosis is most likely?
348. A 24-year-old man has been experiencing pain in the epigastric region for 3 years, occurring 1-1.5 hours after eating and
at night, and heartburn. Three hours ago, the patient noted the appearance of acute pain in the epigastric region, which spread
throughout the abdomen, and difficulty breathing in an upright position. During hospitalization: the patient's condition is of
moderate severity, he is in a position on the right side with his knees bent; body temperature is 36.8, tachycardia is noted up to
90 per minute. What preliminary diagnosis is most likely?
acute appendicitis;
acute pancreatitis;
mesenteric thrombosis;
349. A 46-year-old man underwent gastric resection for a severe penetrating ulcer; 4 days after surgery he was bothered by
abdominal bloating. Peristalsis is not heard, gases do not pass away. The condition is of moderate severity, the skin and visible
mucous membranes are of normal color. In the lungs, breathing is vesicular. Pulse 80 per minute, rhythmic, full, blood pressure
130/90 mm. Hg Art. Heart sounds are muffled. The tongue is a bit dry. The abdomen is evenly swollen, painful in the wound
area. Shchetkin's symptom is negative. The nasogastric tube yields 400 ml of gastric secretion. There are no features from the
urinary system. What type of intestinal obstruction are we talking about?
Dynamic
Mechanical
Spastic
Strangulation
Paralytic
350. What is the recommended first step for controlling external bleeding in surgery?
351. A 57-year-old man was admitted to the clinic with complaints of cough, sputum, sometimes streaked with blood,
weakness, sweating, and an increase in body temperature in the evenings up to 37.5 degrees. C. History: twice within 3 months
the patient was hospitalized in a therapeutic hospital with a diagnosis of right-sided pneumonia. After a course of antibiotic
therapy, he was discharged home with a significant improvement in his condition. On examination: the condition is moderate.
The right half of the chest lags behind during breathing; percussion on the right side shows dullness of the percussion sound,
absence of vocal tremor, and weakening of respiratory sounds. An X-ray examination reveals an expansion of the root shadow of
the right lung and atelectasis of the upper lobe of the right lung. What are the most informative diagnostic methods to clarify
the diagnosis?
4.Bronchography.
5. Examination of sputum.
352. In which of the following immunodeficiency conditions is conjugated H. influenzae. influenzaetype B vaccine
contraindicated?
B-cell disorders
none of this
complement violations
T-cell disorders
NK-cells disorders
353. A 1-month-old, full-term boy is brought to the office for a routine check-up. He is exclusively breastfed. The boy does not
have any medical problems, and his hospital stay after birth was unremarkable. His mother consumes a well-balanced diet,
takes no medications, and does not drink alcohol. The boy is at the 50th percentile for weight and the 60th percentile for height.
His physical examination is normal. At this time, supplementation with which of the following should be recommended for this
infant?
Folic acid
Iron
Vitamin D
Thiamine
Vitamin A
354. A 16-year-old boy is evaluated for excessive skin bruising occurring over the last 6 weeks. There is no personal or family
history of excessive bleeding. The patient has a history of chronic kidney disease and is waiting for a renal transplant.
Laboratory studies are as follows: Hemoglobin-9.5 g/dL Platelets-200,000/mm3 Blood urea nitrogen-50 mg/dL Creatinine-4.4
mg/dL PT-12 sec Activated PTT-23 sec Which of the following is the most likely cause of this patient's bruising?
Consumptive coagulopathy
Dysfunctional platelets
Vitamin K deficiency
355. A 12-month-old boy is brought to the office by his father due to prolonged bleeding from mouth after slipping and hitting
his face on a coffee table. He was born at term, and his mother died shortly after his birth due to bleeding complications
following a home delivery. Vital signs are normal. He is awake and alert. There is blood oozing from his gums and marked
bruises along his trunk and thighs. The remainder of the physical examination is normal. Laboratory evaluation shows
decreased von Willebrand Factor antigen levels and activity, measured via the ristocetin cofactor assay. Which of the following
sets of laboratory values is most likely present in this patient? Platelets PT PTT
356. A 34-week-old baby, weighing 2250 grams at birth, is being followed up in the neonatal intensive care unit due to Coombs
negative hemolytic anemia and thrombocytopenia. While in intensive care, it is observed that the baby's movements are
sluggish, it seems restless and cries when its extremities are held or moved. Since birth, a slight purulent discharge from the
nose and the detection of hepatosplenomegaly are also observed. In the laboratory examinations, elevation in liver function
tests is detected. The cranial ultrasound, reported as normal, shows findings consistent with osteochondritis in the extremity
profiles, including the hand, foot, and knee joints. No cardiac anomaly is reported in the echocardiography. It is learned that
during the mother's pregnancy, she developed ulcerative lesions in the genital area and received antibiotic treatment for this
reason. Which of the following is the most valuable method in the diagnosis of this baby?
VDRL
RPR
FTA-ABS
blood culture
357. A four-day-old male baby, born at 38 weeks weighing 2150 grams, was brought in with decreased breastfeeding, jaundice,
abdominal distension, and rash. On physical examination, the baby had purpuric lesions on the body, jaundice,
hepatosplenomegaly, and an eye examination revealed microphthalmia and chorioretinitis. In the presence of periventricular
calcifications, which of the following is the most likely diagnosis?
Congenital cytomegalovirus
Congenital rubella syndrome
358. A 7-year-old boy is brought to the clinic for a well-child visit. Over the past year, the patient has become fidgety, impulsive,
inattentive, and restless, leading to a progressive decline in his grades. Height and weight are at the 50th and 5th percentiles for
his age, respectively. The patient takes no medications or supplements. The thyroid gland is diffusely enlarged to palpation
without tenderness or bruit. Neurologic examination reveals no focal findings. Results of thyroid function tests are as follows:
TSH-7 mU/L Thyroxine, total-16 µg/dL Triiodothyronine-210 ng/dL (normal: 80-195) Which of the following is the most likely
etiology of this patient's thyroid function test abnormalities?
Graves disease
359. A 6-year-old boy is brought to the emergency department due to bleeding after a dental extraction earlier this morning.
The patient's past medical history is significant for painful swelling of his knee joints after minor trauma. Aspiration of the joints
during several occasions yielded frank blood, and he was diagnosed with hemarthrosis. He has no known allergies. Currently,
hemostasis in this patient most likely can be achieved by the administration of which of the following?
Factor XII
Fibrinogen
Protein C
Thrombin
Urokinase
360. A four-year-old girl who has recurrent urinary tract infections was previously hospitalized and treated for this reason.
Which of the following techniques is the most useful in this child, where renal damage (scar) is suspected?
Uroflowmetry
Micturating cystourethrogram
MRI
361. A four-day-old male baby, born at 38 weeks weighing 2150 grams, was brought in with decreased breastfeeding, jaundice,
abdominal distension, and rash. On physical examination, the baby had purpuric lesions on the body, jaundice,
hepatosplenomegaly, and an eye examination revealed microphthalmia and chorioretinitis. Which of the following should
primarily be considered in this infant?
Early sepsis
Late sepsis
Viral sepsis
Congenital infection
birth defect
362. A 12-year-old girl comes to the office with constant swelling and pain of her elbows for the past week that have prevented
her from participating in basketball practice. She also had knee pain during the preceding week that was attributed to a fall
during practice. Her parents say that she is healthy and has had only minor illnesses that children typically experience during
the winter. The patient's temperature is 38.9 C , blood pressure is 110/70 mm Hg, and pulse is 110/min. Her elbows are swollen
and tender with limited range of movement. Her knees appear normal. A new holosystolic murmur is heard on cardiac
auscultation. Antistreptolysin O titers are 400 Todd units/mL (normal: <300 Todd units/mL). The patient is admitted to the
hospital. During her hospitalization, this patient is at greatest risk of dying from which of the following complications?
Mitral stenosis
Pancarditis
Renal failure
Septic arthritis
Septic shock
363. Your patient is a 7-year-old girl who was prescribed an antibiotic for an ear infection. After 4 days of taking the antibiotic,
the patient developed hives and itching on the face, throat and neck. Which of the following should you do?
Tell your parents that these symptoms are related to an ear infection, and the antibiotic will eliminate these symptoms if taken
as directed.
Provide supportive care and monitor the patient's condition during transportation to the hospital.
364. A newborn, brought in with umbilical cord bleeding, has normal prothrombin time and partial thromboplastin time.
Considering that the parents are related and there is a sibling with similar findings who was lost, what is the most likely
diagnosis for this patient?
Factor V deficiency
C3b
IL-2
Histamine
Lysozyme
TNF-α
366. A four-day-old male baby, born at 38 weeks weighing 2150 grams, was brought in with decreased breastfeeding, jaundice,
abdominal distension, and rash. On physical examination, the baby had purpuric lesions on the body, jaundice,
hepatosplenomegaly, and an eye examination revealed microphthalmia and chorioretinitis. Which of the following is the most
common long-term sequel of congenital cytomegalovirus infection?
Hydrocephalus
Hearing loss
Chorioretinitis
Bone deformation
367. A 9-year-old girl is brought to the emergency department due to prolonged epistaxis. The girl says that she picked her
nose immediately before the bleeding started. Her parents decided to bring her to the emergency department after the epistaxis
persisted for 20 minutes despite constant compression of the nasal alae. The patient has had frequent nosebleeds that often
last >10 minutes. Her family history is significant for a grandfather who had an unspecified bleeding disorder. Given the history
of prolonged, recurrent nosebleeds, laboratory tests are ordered, and results are as follows: Hematocrit-43% Bleeding time-
prolonged Partial thromboplastin time (PTT)-prolonged Prothrombin time (PT)-normal Thrombin time (TT)-normal D-dimer-
normal Which of the following is the most likely diagnosis?
Dysfibrinogenemia
Hemophilia A
368. A newborn girl is admitted to the neonatal intensive care unit with jaundice, hepatomegaly, and generalized edema. She
was born by vaginal delivery to a 32-year-old woman, gravida 2, para 2, who received minimal prenatal care. The infant's
laboratory results show a hemoglobin level of 6 g/dL and a positive direct Coombs test. A peripheral blood smear shows many
nucleated erythrocytes. The infant has significant respiratory distress due to pleural effusions and ascites and dies soon after
birth. Autopsy shows areas of extramedullary hematopoiesis in many tissues. Which of the following is the most likely cause of
this patient's condition?
369. A 10-year-old boy who recently immigrated from overseas with his family is brought to the office due to exertional dyspnea
and fatigability. The boy tires easily when walking and cannot keep up with his peers at the playground. According to his
parents, he was diagnosed with congenital heart disease in infancy, but corrective treatment was unavailable. They cannot
recall the details of his diagnosis. He takes a daily multivitamin and no medications. Pulse oximetry shows 98% in the right
hand and 84% in the right foot. Physical examination reveals no murmurs. There is bilateral cyanosis and clubbing of the toes.
The hands and fingers appear normal. All extremity pulses are full and equal. Which of the following is the most likely
diagnosis?
Tetralogy of Fallot
370. A 2-day-old boy is evaluated in the newborn nursery due to vomiting. He was born at term to a 30-year-old primigravida via
spontaneous vaginal delivery. The patient initially tolerated formula after birth. However, the past 2 feeding attempts resulted in
bilious emesis. Cardiopulmonary examination is unremarkable. Abdominal examination shows distension with active bowel
sounds. There is no hepatosplenomegaly. Digital rectal examination results in a forceful expulsion of gas. Which of the
following processes was most likely impaired during embryologic development of this patient's gastrointestinal system?
Cell migration
Obliteration
Recanalization
Rotation
Vascularization
371. On a hot summer day, you are called to a local park to see an 8-year-old child with respiratory failure. The child can barely
speak and seems sleepy. Vital signs are blood pressure 90/60, pulse 120, and respiratory rate 32. Physical examination reveals
a swollen tongue, stridor when inhaling, wheezing, and hives on the neck and chest. What condition do you suspect?
Allergic reaction
Asthma
Anaphylaxis
Heat exhaustion
Angioedema
372. A patient with delayed meconium discharge at birth after 48 hours. What is the initial research?
barium test
rectal manometry
rectal biopsy
colonoscopy
373. Your patient ate a casserole that might have included seafood. The patient is concerned because he is allergic to seafood.
When examined, you don't show any signs or symptoms of anaphylaxis. The patient's vital signs are stable, and he is prescribed
an automatic epinephrine injector. Which of the following is most appropriate?
Inform the patient that they do not currently need treatment or transportation.
374. A 12-year-old girl is brought to the clinic by her parents after she is found to have hypertension by her school nurse. The
patient has no symptoms and reads a book during the office visit. The patient has had several episodes of fever and abdominal
pain, which her parents treated with over-the-counter antibiotics. Blood pressure is elevated on several readings in the office.
There is no family history of hypertension. Renal ultrasound reveals dilated calyces with overlying cortical atrophy bilaterally,
mostly in the upper and lower poles. Which of the following is the most likely cause of this patient's condition?
Malignant hypertension
Reflux nephropathy
375. A term newborn is evaluated for cyanosis immediately after birth. On examination, the patient's oxygen saturation is 70% in
all 4 extremities and does not change despite 100% oxygen administration. The lungs are clear to auscultation and there are no
heart murmurs or gallops. Peripheral pulses are normal. A chest radiograph reveals clear lungs and a normal cardiac silhouette
but a narrowed mediastinal shadow. Which of the following is the most likely cause of cyanosis in this patient?
Bulbus cordis
Primitive atria
Sinus venosus
377. A 13-year-old boy is evaluated for abnormal growth. His mother says that he has grown rapidly over the past 10 months
and is much taller than his older brother and other children his age. During this time, she has had a hard time finding clothes
that fit him and has had to buy new shoes for him each month due to increasing foot size. The patient says that he feels well
except for excessive sweating. He has no history of major medical illness and takes no medications. Both parents are of
average height and have normally proportioned limbs. The patient is above the 99th percentile for height and at the 90th
percentile for weight. On physical examination, he has long extremities with large hands and feet; the lower jaw is protruding.
Heart sounds are normal. Sexual development is Tanner stage 4. Which of the following is the most likely underlying cause of
this patient's condition?
378. A 15-year-old boy is evaluated for fever, headache, malaise, and cough 3 weeks after returning from summer camp. Lung
auscultation is unremarkable. Chest x-ray reveals nodular infiltrates. The patient's blood samples are collected for analysis in
anticoagulated tubes, which are transported submerged in ice; when taken out several minutes later, turbidity and clumping are
noted. Warming the tube to body temperature leads to rapid dissolution of the clumps. Which of the following organisms is
most likely responsible for this patient's condition?
Coccidioides immitis
Coxiella burnetii
Haemophilus influenzae
Histoplasma capsulatum
Mycoplasma pneumoniae
379. An 8-year-old girl is brought to the emergency department due to worsening shortness of breath and chest tightness for
the past 2 hours. On physical examination, the patient speaks in short sentences. Lung auscultation reveals diffuse wheezing.
During the evaluation, the patient is instructed to take a deep breath and blow as hard as possible into the mouthpiece of a
hand-held device. The measured airflow rate is significantly lower than expected. Treatment with an inhaled medication is
begun. A repeat evaluation several minutes later shows an increase in the airflow rate. The administered medication most likely
affected which of the following to produce the observed finding in this patient?
Airway inflammatory response
380. A 14-year-old boy is brought to the emergency department due to excessive urination and thirst. He has lost 4.5 kg in the
last 3 weeks. The patient has no past medical problems. His father has type 1 diabetes mellitus. Physical examination shows
dry mucous membranes. Laboratory studies reveal blood glucose of 455 mg/dL, normal anion gap, and hemoglobin A1c of
11.3%. The patient is diagnosed with type 1 diabetes, and treatment with insulin is initiated. In addition to lowering blood
glucose, insulin increases glycogen synthesis in hepatocytes. Activation of which of the following molecules most likely
promotes this metabolic effect?
Lipoxygenase
Phospholipase C
Protein kinase A
Protein phosphatase
381. What of the following would you expect to hear when auscultating the lungs of a patient with an anaphylactic reaction?
Wheezing sounds
rough-sounding ronkies
Stridor
382. A 7-year-old boy is brought to the clinic for follow-up care. The patient recently completed a course of antibiotics for a
urinary tract infection caused by Escherichia coli. He has had 2 febrile urinary tract infections in the past, both of which resolved
following antibiotic therapy. Vital signs are normal. Examination of the abdomen is unremarkable. Renal ultrasound examination
reveals no signs of hydronephrosis. Voiding cystourethrogram shows contrast filling the bladder and extending linearly toward
the umbilicus. This patient most likely has a congenital abnormality related to which of the following structures?
Omphalomesenteric duct
Umbilical artery
Umbilical vein
Urachus
Ureter
383. An 8-year-old girl is brought to the emergency department due to "not feeling well." The patient was at an outdoor picnic
and began experiencing nausea, vomiting, abdominal cramps, and watery diarrhea 30 minutes ago. She also reports feeling
dizzy. The patient has no prior medical conditions and takes no medications. Temperature is 37 C , blood pressure is 60/30 mm
Hg, pulse is 140/min, and respirations are 28/min. On physical examination, the patient appears pale and listless. There is faint
bilateral wheezing. The abdomen is soft and nontender. Scattered wheals are present. Which of the following is most
responsible for this patient's current condition?
Kallikrein-generated bradykinin
384. A 2-month-old boy is brought to the emergency department due to progressively worsening "floppiness" and poor feeding.
The infant was born via an uncomplicated vaginal delivery to a 38-year-old woman. The parents describe the infant as a "good
baby" who rarely cries and sleeps through the night but has lately been difficult to rouse for breastfeeding. Stools have also
decreased to every other day and are small and pellet-like. Physical examination shows a hypotonic infant with a large anterior
fontanelle, large tongue, and a reducible umbilical hernia. He has low tone and is unable to hold his head erect on his own. No
other abnormalities are seen. Which of the following is the most likely cause of this patient's condition?
Botulism
Down syndrome
Galactosemia
Hirschsprung disease
Hypothyroidism
385. Which of the following disorders is the result of a factor VIII deficiency?
Christmas Disease
Hemophilia B
Hemophilia A
386. A 5-year-old boy is brought to the emergency department due to 2 days of dark, low-volume urine and decreased energy.
The parents say that the boy had abdominal pain, fever, and bloody diarrhea for 4 days, which resolved about 3 days ago without
treatment. The day before the onset of symptoms, the patient swam in a lake and ate hamburgers at a family picnic. His vital
signs are within normal limits. Physical examination shows pallor but is otherwise normal. He has no peripheral edema or
rashes. Laboratory evaluation shows anemia, thrombocytopenia, and elevated blood urea nitrogen and serum creatinine. Which
of the following findings is most likely to be seen in this patient?
387. A 4-month-old girl is diagnosed with a febrile urinary tract infection. A voiding cystourethrogram shows retrograde urine
flow into the right ureter but no associated ureteral dilation. One year later, a voiding cystourethrogram is repeated and shows
no abnormalities. Which of the following is the most likely cause of this infant's presentation?
Renal agenesis
388. A 3-month-old boy is brought to the office due to fussiness, poor weight gain, and polyuria. Urine volume is 700-800
mL/day. The patient's maternal grandfather has polydipsia and polyuria. Serum sodium is 151 mEq/L. Genetic testing reveals a
vasopressin-2 receptor mutation. Which of the following additional findings is most likely to be seen in this patient? Serum
osmolality Urine osmolality after water deprivation Change in urine osmolality with desmopressin administration
389. The presence of a tracheoesophageal fistula (TEF) in addition to esophageal atresia leads to what characteristic symptom?
Biliary vomiting
Failure to thrive
vomiting
390. A 12-year-old boy has prolonged oral bleeding immediately after a tooth extraction. Despite several interventions, the
bleeding persists for hours and stops only after desmopressin (DDAVP) administration. The patient has several dental caries
due to excessive juice intake and inconsistent oral hygiene. He has no other medical problems and takes no medications. His
father and paternal grandmother have also had excessive bleeding after dental procedures. Review of systems is positive for
mild bruising on his legs. Which of the following is the most likely therapeutic mechanism of desmopressin for this patient's
condition?
391. A 1-hour-old boy in the newborn nursery is evaluated for jitteriness and irritability. The neonate was born by cesarean
delivery due to arrest of the second stage of labor. The patient has not yet breastfed as his mother is still recovering from
surgery. The mother's pregnancy was complicated by gestational diabetes mellitus; she was prescribed insulin but did not take
it regularly. Birth weight is 4.5 kg . Cardiopulmonary examination is within normal limits. Serum glucose is 21 mg/dL. Which of
the following is the most likely cause of this neonate's hypoglycemia?
392. A newborn boy is found to have a serum TSH above and serum thyroxine below the normal range for his age. The patient
was born at 39 weeks gestation via spontaneous vaginal delivery. His mother has no medical conditions. On examination, the
patient's length and weight are normal, and he has no apparent congenital abnormalities. Ultrasound of the neck shows a
normal-sized thyroid gland in the appropriate location. Which of the following is the most likely cause of this patient's
condition?
Central hypothyroidism
393. A four-day-old male baby, born at 38 weeks weighing 2150 grams, was brought in with decreased breastfeeding, jaundice,
abdominal distension, and rash. On physical examination, the baby had purpuric lesions on the body, jaundice,
hepatosplenomegaly, and an eye examination revealed microphthalmia and chorioretinitis. Which of the following is the most
common clinical condition caused by congenital rubella syndrome?
Microcephaly
Retinopathy
Hearing loss
Pulmonary stenosis
Bone deformation
394. A 10-year-old boy is brought to the physician by his parents due to restlessness and involuntary jerking. He takes no
medications and his vaccinations are up-to-date. His parents do not recall any recent injuries or illnesses other than a sore
throat 3 months ago. On examination, the patient has rapid, irregular jerking movements involving his face, arms, and legs. This
patient is at greatest risk for developing which of the following conditions?
Deforming polyarthritis
Early dementia
Parkinson disease
Renal failure
395. A 7-year-old boy comes to the office for follow-up. A week ago, the patient began having episodes of bloody diarrhea that
have since resolved. His current laboratory studies show elevated blood urea nitrogen and serum creatinine levels. Complete
blood count reveals anemia and thrombocytopenia. Coagulation studies are within normal limits. His peripheral blood smear is
shown in the image below. Which of the following is the most likely cause of this patient's anemia?
Erythropoietin deficiency
396. A 2-hour-old newborn has a blood glucose concentration of 30 mg/dL. The patient was delivered via caesarean to a 32-
year-old woman at 38 weeks gestation. Birth weight is 4.5 kg (9 lb 14 oz). The pregnancy was complicated by gestational
diabetes treated with insulin. However, glycemic control remained suboptimal due to maternal noncompliance with insulin
treatment and poor adherence to dietary recommendations. Examination of the neonate is unremarkable. Which of the following
is the most likely primary mechanism responsible for this patient's low blood glucose concentration?
Hyperfunctioning pancreas
Insulin resistance
397. A 3-month-old girl is brought to the emergency department due to fever, irritability, and vomiting for the past 2 days. On
examination, she is ill-looking, lethargic, and febrile. Blood cultures grow Mycobacterium tuberculosis. One of her brothers died
from disseminated mycobacterial infection during infancy. Impairment of which of the following protective mechanisms is most
likely contributing to this patient's infection?
Antibody production
Complement production
Interferon signaling
Isotype switching
Leukocyte adhesion
398. A 2-year-old girl is brought to the emergency department with intermittent pain in the abdomen that began several hours
ago. The patient also had a bowel movement that appeared tinged with blood. Ultrasound shows an area along the ileum in
which the proximal small intestine is telescoped into the distal small intestine. Reduction via enema is unsuccessful, and the
patient undergoes laparotomy with resultant resection of the involved intestine. The pathologic finding at the lead point is
shown in the exhibit. Which of the following is the most likely underlying diagnosis in this patient?
Burkitt lymphoma
Henoch-Schönlein purpura
Meckel diverticulum
Pseudomembranous colitis
399. A 6-week-old infant is brought to the office due to difficulty feeding. The patient was born at 39 weeks gestation via
spontaneous vaginal delivery. He is breastfeeding for 5 minutes per side every 3 hours. The patient has profuse sweating and
difficulty breathing with feeds. Weight is at the 5th percentile. On examination, the patient is tachypneic. Cardiac examination
shows a holosystolic murmur at the left lower sternal border; a palpable thrill is present at the left sternal border. Compared to a
healthy infant, this patient likely has which of the following intracardiac pressure changes? Right atrium Right ventricle. Left
atrium. Left ventricle
400. Which of the following is included in the initial assessment of a conscious patient suffering from anaphylactic shock?
Finding out how the patient was exposed to the substance to which she is allergic
Determination of whether the patient's systolic blood pressure exceeds 100 mm Hg.
401. A term newborn boy is evaluated in the neonatal intensive care unit for respiratory distress. Apgar scores are 2 and 5 at 1
and 5 minutes, respectively. Respirations are 84/min. On examination, the patient has a barrel chest with a scaphoid abdomen
and mild cyanosis of his extremities. Auscultation shows absent breath sounds on the left; right lung aeration is normal. Chest
x-ray reveals multiple fluid-containing cystic areas on the left and a mediastinal shift to the right. Which of the following
embryologic events most likely failed to occur in this patient?
402. 9-month-old child with cough, shortness of breath, subfebrile body temperature was examined by a local pediatrician.
Focal bronchopneumonia was suspected. What auscultation picture is typical for this case?
Rough breathing
Local rales
dry wheezing
403. A 10-year-old boy comes to the office for a first visit. The patient's mother says that he has required several blood product
transfusions due to anemia, but she does not have prior records available with her. On examination, the patient's temperature is
37.1 C (98.8 F). BMI is 21 kg/m2. Examination is notable for conjunctival pallor and moderate splenomegaly. Laboratory results
are as follows: Hemoglobin-9.4 g/dL Platelets-240,000/mm3 Enzyme assays performed on circulating blood cells demonstrate
low pyruvate kinase activity. Which of the following is the most likely cause of this patient's splenomegaly?
Inflammatory infiltration
Myeloproliferative disorder
404. Researchers studying gastrointestinal pathophysiology analyze hundreds of gastric mucosal biopsy specimens taken from
patients who underwent endoscopy at a local tertiary care center. They notice that colonization of the gastric antrum with S-
shaped, gram-negative bacteria is associated with a decreased number of somatostatin-producing antral cells. Depletion of
these cells from the gastric antrum is most likely to cause which of the following conditions?
Chronic pancreatitis
Duodenal ulcers
Gastric lymphoma
Gastric ulcers
405. A newborn boy is found to have elevated TSH and low thyroxine (T4) levels on routine newborn screening. The patient was
born vaginally to a 32-year-old, healthy woman at 39 weeks gestation. The 5-minute Apgar score was 9. Ultrasound of the neck
reveals an ectopically located, small thyroid gland just above the hyoid bone. Early treatment with levothyroxine is indicated
primarily to prevent which of the following?
Neurocognitive dysfunction
Nutrient malabsorption
Proptosis
406. A 3-day-old boy is brought to the emergency department due to poor feeding, emesis, and lethargy over the past 24 hours.
He was born via uncomplicated spontaneous vaginal delivery to a 30-year-old woman who had a normal pregnancy. The boy
was discharged from the newborn nursery yesterday and was breastfeeding exclusively until the onset of symptoms. Stool and
urine output were normal while he was in the hospital. The patient is afebrile and normotensive but tachycardic and tachypneic.
He appears dehydrated, and the abdomen is distended. The patient vomits during the examination; the vomitus is shown in the
exhibit. On laparotomy, fibrous bands are seen extending from the cecum and right colon to the retroperitoneum, causing
extrinsic compression of the duodenum. Which of the following embryologic processes most likely failed in this patient?
150-200 IU
400-500 IU
1000-1500 IU
2000-5000 IU
2500- 5000 IU
408. A peptic ulcer is an erosion in the stomach or the first few centimeters of the duodenum. Nearly all ulcers are caused by
Helicobacter pylori infection or use of nonsteroidal anti-inflammatory drugs. Several risk factors exist for the development of
ulcers and their complications. Which of the following risk factors impairs healing and increases the incidence of recurrence?
Alcohol use
Cigarette smoking
History of gastrinoma
Spicy food
409. A 3-year-old boy diagnosed with thalassemia major receives multiple transfusions to maintain his blood hemoglobin level.
On physical examination, his liver and spleen are mildly enlarged. He also has glucose intolerance as demonstrated by an oral
glucose tolerance test. Which of the following therapies would prevent congestive cardiac failure in this patient?
Erythropoietin
Cobalamin
Deferoxamine
Ascorbic acid
Digoxin
410. Which of the following diagnostic methods allow to assess the functional state of the gallbladder in children with
dyskinesia?
Еsophagogastroduodenoscopy
Ultrasound of gallbladder
411. A 14-year-old boy is brought to the emergency department due to excessive urination and thirst. He has lost 4.5 kg in the
last 3 weeks. The patient has no past medical problems. His father has type 1 diabetes mellitus. Physical examination shows
dry mucous membranes. Laboratory studies reveal blood glucose of 455 mg/dL, normal anion gap, and hemoglobin A1c of
11.3%. The patient is diagnosed with type 1 diabetes, and treatment with insulin is initiated. In addition to lowering blood
glucose, insulin increases glycogen synthesis in hepatocytes. Activation of which of the following molecules most likely
promotes this metabolic effect?
Lipoxygenase
Phospholipase C
Protein kinase A
Protein phosphatase
412. A 6-year-old girl is brought to the office by her mother due to abdominal pain. The pain is diffuse and began this morning; it
waxes and wanes in intensity. The patient has had 2 episodes of nonbilious emesis. She has had no fevers, sore throat, diarrhea,
or bloody stools. Two days ago, the patient developed an erythematous, macular rash over her legs and back that has become
darker and more confluent today. Blood pressure is 95/60 mm Hg. The abdomen is soft and there is diffuse, mild tenderness on
palpation with no rebound, guarding, or appreciable masses. A raised, nonblanching rash is noted over the legs and back.
Urinalysis results are as follows: Specific gravity--1.022Protein-+1 Blood-+3 Glucose-negative Ketones-negative Leukocyte
esterase-negative Nitrites-negative Which of the following renal abnormalities is most likely to be present in this patient?
414. I n contrast to acute renal failure, which of the following signs are the most reliable signs of CKD?
metabolic alcalosis
415. A 13-month-old boy is brought to the emergency department due to cough and increased work of breathing. He has a
history of recurrent otitis media, pneumonia, and thrush as well as chronic diarrhea and failure to thrive. On physical
examination, he is tachypneic and has perioral cyanosis. A chest radiograph shows bilateral interstitial opacities. The patient is
admitted to the hospital and undergoes bronchoscopy. Analysis of the bronchoalveolar lavage fluid reveals Pneumocystis
jirovecii. Which of the following is the most likely underlying diagnosis?
Agammaglobulinemia
Cystic fibrosis
416. A four-day-old male baby, born at 38 weeks weighing 2150 grams, was brought in with decreased breastfeeding, jaundice,
abdominal distension, and rash. On physical examination, the baby had purpuric lesions on the body, jaundice,
hepatosplenomegaly, and an eye examination revealed microphthalmia and chorioretinitis. In the case of the patient having
hydrocephalus and widespread cerebral calcifications, which of the following is the most likely diagnosis?
Congenital cytomegalovirus
417. A four-day-old male baby, born at 38 weeks weighing 2150 grams, was brought in with decreased breastfeeding, jaundice,
abdominal distension, and rash. On physical examination, the baby had purpuric lesions on the body, jaundice,
hepatosplenomegaly, and an eye examination revealed microphthalmia and chorioretinitis. Which of the following congenital
infections is most commonly expected in this infant?
Congenital cytomegalovirus
418. An 18-month-old boy is brought to the physician by his parents for fever, runny nose, and sore throat. The physician
reassures the parents and recommends supportive care with plenty of fluids. He sends them home with instructions to follow
up if the boy’s symptoms worsen. Two days later, the infant is brought to the emergency department with persistent fever,
brassy cough, and difficulty breathing. Physical examination reveals stridor. Which of the following pathogens is most likely
responsible for this patient's condition?
Rhinovirus
Paramyxovirus
Togavirus
Parvovirus
Calicivirus
419. A 9-month-old girl is brought to the emergency department due to fever and cough. Her symptoms began approximately 2
days ago and have been worsening. The patient is otherwise healthy except for an episode of acute otitis media 3 weeks ago
that resolved following a course of oral amoxicillin. She lives with her mother, father, and grandmother who have been in good
health. Her mother smokes cigarettes. Temperature is 38.5 C (101.3 F) and respirations are 34/min. Pulse oximetry is 95% on
room air. Pulmonary examination reveals mild subcostal retractions and focal crackles in the left lower lobe. Which of the
following underlying factors is most likely contributing to this patient's current condition?
420. A 34-week-old baby, weighing 2250 grams at birth, is being followed up in the neonatal intensive care unit due to Coombs
negative hemolytic anemia and thrombocytopenia. While in intensive care, it is observed that the baby's movements are
sluggish, it seems restless and cries when its extremities are held or moved. Since birth, a slight purulent discharge from the
nose and the detection of hepatosplenomegaly are also observed. In the laboratory examinations, elevation in liver function
tests is detected. The cranial ultrasound, reported as normal, shows findings consistent with osteochondritis in the extremity
profiles, including the hand, foot, and knee joints. No cardiac anomaly is reported in the echocardiography. It is learned that
during the mother's pregnancy, she developed ulcerative lesions in the genital area and received antibiotic treatment for this
reason. In the clinical diagnosis of this baby, the likelihood of the microorganism causing the infection passing to the fetus is
highest during which period of the mother's pregnancy?
First trimester
Second trimester
3rd trimester
Staphylococcus aureus
Neisseria
Candida albicans
Hemophilic influenza
taphylococcus epidermis
422. A 7-year-old boy is brought to the office due to sudden onset of facial swelling 2 hours ago. He has had no itching or pain
other than a sore throat over the last 2 days, for which he has taken acetaminophen. The patient has had similar episodes of
facial swelling that resolved spontaneously after a few days. Temperature is 37 C , blood pressure is 100/78 mm Hg, pulse is
95/min, and respirations are 24/min. Examination shows nonpitting edema of the cheeks, lips, and tongue; there is no
tenderness or erythema. Which of the following studies is most likely to be abnormal?
Eosinophil count
Serum C4 level
Serum C8 level
423. A 6-hour-old boy is in the newborn nursery with feeding difficulties. The patient was born at 39 weeks gestation to a 33-
year-old primigravida via cesarean delivery due to failure to progress and late decelerations seen on fetal heart tracing. Apgar
scores were 8 and 9, but examination shows an infant with excessive drooling and coughing. Cardiac, respiratory, and
abdominal examinations are otherwise normal at rest. When the infant attempts to breastfeed, however, several bouts of
coughing and perioral cyanosis develop with oxygen saturation of 85% on room air. Which of the following is the most likely
cause of this patient's condition?
Chest X-ray
Echocardiogram
Electrocardiogram (EСG)
425. Human papillomavirus vaccination is recommended for which of the following age groups?
26 to 40 years
Over 40 years
9 to 26 years
1 to 5 years
6-9 years
426. Which of the following is the most important physical examination for sports participation in adolescents?
Neurological exam
Respiratory exam
Cardiac exam
Genitourinary exam
Abdominal exam
427. A 40-year-old man, suffering from peptic ulcer syndrome for a long time, has recently noted the disappearance of pain
cyclicity, the pain has become constant, shingles. Objectively: condition of average severity. The abdomen is moderately
bloated, pain is determined in the epigastric region and in the Shoffar zone. Which of the following conditions is most likely to
have occurred?
Ulcer perforation
Gatekeeper stenosis
Sputum examination.
429. Which of the following is the most common histologic type of cancer of the lung?
Adenocarcinoma
Neuroendocrine carcinoma
430. You are receiving shift report on a patient with cirrhosis. The nurse tells you the patient’s bilirubin levels are very high.
Based on this, what assessment findings may you expect to find during your head-to-toe assessment? Select all that apply:
431. Which of the following is suggestive of the window period of hepatitis B virus infection?
432. What is the most appropriate age for discontinuing cervical cancer screening in average-risk patients who have adequate
prior screening and no factors that warrant extended screening?
65 years
75 years
55 years
35 years
45 years
433. Which of the following A1C values is most appropriate in most adults with type 2 diabetes?
≤ 6.0 percent
≤ 7.5 percent
≤ 5.5 percent
≤ 6.5 percent
≤ 7.0 percent
434. This is a 47-year-old male with cough and fever. From the appearance of this disease, which organism would be the most
likely etiologic agent?
Tuberculosis
Pneumocystic pneumonia
Staphylococcal pneumonia
Aspergillosis
Pneumococcal pneumonia
435. A 30-year-old male patient suffering from type I diabetes mellitus was found in a comatose state 3 hours after insulin
administration. What activity needs to be completed first?
Take an ECG
436. During a cardiac arrest, a 58-year-old man, a non-smoker, receives cardiopulmonary resuscitative measures and is brought
to the hospital, where he is intubated. During the intubation procedure he suffers aspiration of gastric contents. Over the next
10 days he develops a non-productive cough along with a fever to 37.9°C. A chest radiograph reveals a 4 cm diameter mass
with an air-fluid level in the right lung. A sputum gram stain reveals mixed flora. Which of the following conditions is he most
likely to have?
Lung abscess
Chronic bronchitis
Bronchiectasis
COPD
437. Which of the following is NOT an appropriate component of a family physician's scope of care?
Biopsychosocial model
438. Which of the following symptoms is more suggestive of influenza rather than the common cold?
Sore throat
Nasal congestion
Myalgia
Sneezing
Postinfarction cardiosclerosis
Unstable angina
COPD
Hypertension
Diabetes mellitus
440. Which of the following patients is least likely to benefit from screening for diabetes mellitus?
441. A 53-year-old patient, chronic cough, dyspnea, and decreased breathing, smoke for more than 30 years. Probable
diagnosis?
Asthma
Lungs' cancer
Bronchiectasis
Pneumonia
442. Which medication is typically recommended to prevent arterial thromboembolism in patients with artificial heart valves?
Alteplase
Clopidogrel
Warfarin
Dabigatran
Prasugrel
443. Which of the following interventions is most likely to prevent recurrences of low back pain?
Acupuncture
Exercise
Massage therapy
Tricyclic antidepressant
Obesity
Morbid obesity
History of pancreatitis
History of hypoglycemia
445. From whom is the history generally obtained for infants and preschool children?
Patient's relatives
Patient's caregiver
Mother's obstetrician
Patient
Patient's siblings
447. Which of the following statements accurately describes the pathogenesis of pernicious anemia?
Folate deficiency
It is a gram-negative bacterium
Headache
Joint pain
Productive cough
Nausea
Rapid breathing
450. Which drug is NOT typically used in the therapy of heart failure?
Diuretics
Beta blocker
Statins
Glucocorticoids
452. Risk factors for the development of coronary heart disease do NOT include:
alcohol consumption
nephroptosis
smoking
hypodynamia
obesity
453. Which of the following describes the best course of action for the management of acute liver failure?
Paracentesis
Liver biopsy
Colonoscopy
Endoscopy
454. An 18-year-old woman who’s diagnosis is Diabetes Mellitus type 1, 3 months ago went for examination to the doctor. Her
initial HbA1c level was 12.3%. She was started on a basal and prandial insulin regimen (insulin Glargine and insulin Aspart). Her
blood glucose readings now show episodes of both fasting and postprandial hypoglycemia, with levels ranging from 45 to 60
mg/dL. Physical examination: Blood pressure is 115/60 mm Hg, pulse is 60 beats/min, and BMI is 18 kg/m2. Laboratory test
result: Her current HbA1c level is 6.2%. Which of the physical examination findings would most likely be present in the patient
described above?
Temperature intolerance
Sunken orbits
Increased energy
455. Ventricular septal defect has which of the following types of murmurs?
Still’s murmur
funnel-shaped;
normal;
barrel-shaped;
scoliotic;
scaphoid.
Is physiologically occurring
458. Aortic stenosis directly affects which of the given chambers of the heart?
Atriums
Left atrium
Left ventricle
Right ventricle
Right atrium
Hepatitis B
Hypertension
Diabetes mellitus
Osteoporosis
Migraine headaches
460. According to Starling's law, what effect does increase end diastolic volume have within a healthy heart?
461. Right ventricular hypertrophy is most likely to be present in which of the following conditions?
Ebstein Anomaly
Coarctation of aorta
Aortic stenosis
Truncus arteriosus
462. This is a 78-year-old female with chest pain. This radiograph demonstrates the typical appearance of which of the
following:
463. Which of the following is NOT known as a risk factor for venous thromboembolism?
Pregnancy
Thalassemia
Cancer
464. In case of paroxysm of supraventricular tachycardia, all of the listed remedies can be used, except:
Electrical discharge
465. What is the primary chronic heart disease, manifested by severe hypertrophy of the left ventricular myocardium in the
absence of aortic stenosis, high blood pressure with mandatory massive hypertrophy of the interventricular septum?
dilated cardiomyopathy
hypertrophic cardiomyopathy
restrictive cardiomyopathy
myocarditis
466. Which of the following statements are true of Coarctation of the aorta?
It usually presents with cyanosis and can be detected with a drop in oxygen saturations from pre to post ductal measurements
It can present with renal failure and lactic acidosis as the duct closes
467. A 35-year-old man consulted a doctor with complaints of pain in the upper abdomen that occurred immediately after
eating for 2 months. The pain is sharp, localized in the epigastrium, and does not radiate. He reports eating less frequently to
avoid pain and has lost 4 kg of weight during this time. The patient has smoked a pack of cigarettes per day for 20 years and
drinks 3 bottles of beer per day. His vital signs are within normal limits. Height 165 cm, weight 76.6 kg; BMI 28 kg/m2. Physical
examination - mild tenderness in the upper abdomen without tension. Bowel sounds are normal. Laboratory tests are within
reference ranges. This patient is at greatest risk for which of the following conditions?
Malignant transformation
Gastric perforation
Pseudocyst formation
Gastrointestinal bleeding
469. What is the primary pharmacological action of angiotensin receptor blockers (ARBs)?
470. Which of the following conditions will result in a mild increase in HbA2, mild decrease in HbA1, a normal fetal hemoglobin,
and absent Hb H on electrophoresis?
Hb H disease
Normal hemoglobin
Alpha-thalassemia
Beta-thalassemia trait
Beta-thalassemia major
pulmonary hypertension
low systolic BP
472. Which of the following are preferred screening tests for type 2 diabetes in the general population?
474. The patient is a 43-year-old male with pneumonia. Based on the frontal chest radiograph, the pneumonia is in the:
475. In addition to a beta-blocker and an angiotensin-converting enzyme inhibitor, which of the following medications is most
appropriate in a heart failure patient that remains symptomatic for heart failure and has NOT attained goal blood pressure?
Furosemide
Hydrochlorothiazide
Prazosin
Amlodipine
Spironolactone
Beta-adrenoblockers
ACE-inhibitors
Loop diuretics
Thiazide diuretic
477. Dilated esophagus with a "bird's beak" appearance on barium radiography is consistent with which of the following
conditions?
Achalasia
Hiatal hernia
Nutcracker esophagus
478. Which of the following is generally NOT recommended in the treatment of acute low back pain?
Spinal manipulation
Physical therapy
Acetaminophen
Bedrest
479. When an elderly patient is brought in by a family member who is concerned about their weight loss, what is the most
appropriate next step in assessment of the patient?
480. What is the primary purpose of palpation of the scrotum in the genital examination of a boy?
481. Which of the following lung volumes will increase significantly when air trapping occurs in the lungs of patients with
asthma?
Tidal volume
Residual volume
Vital capacity
482. Which of the following is the most appropriate diagnostic evaluation in patients who are suspected of having secondary
hypertension and do NOT have primary kidney disease, primary aldosteronism, or pheochromocytoma?
Evaluation for renovascular hypertension
Cardiac angiography
Renal biopsy
483. Which of the following most commonly causes inadequate dietary intake leading to poor weight gain?
Genetic factors
Economic factors
Organic diseases
Legal factors
Psychosocial factors
485. Which of the following is most likely to result in acute liver failure?
Hemochromatosis
Hepatic cyst
Acetaminophen toxicity
Bradycardia
Vesicular rash
Abdominal rebound tenderness
487. Which of the following medications can increase serum creatinine without a change in overall kidney function?
Valproate
Atorvastatin
Gentamicin
Naproxen
Trimethoprim
488. Which of the following presentations is associated with large airway obstruction due to lung cancer?
Cough
Weight loss
Hemoptysis
Fatigue
6 months
1 month
1 year
3 months
9 months
490. Which of the following statements regarding antinuclear antibodies (ANA) is MOST accurate?
491. Which of the following drugs is used for the prevention of thromboembolism in the case of atrial fibrillation?
Acetylsalicylic acid
Warfarin
Eptifibatide
Clopidogrel
Tirofiban
492. Which medication is recommended to reduce the risk of cardiovascular events in patients with type 2 diabetes and high
cardiovascular risk?
Metformin
Sulfonylurea
Rosiglitazone
Acetylsalicylic acid
Warfarin
Testicular cancer
Pneumonia
Cervical cancer
Obesity
494. A 45-year-old man who was diagnosed with diabetes 3 months ago presents for follow-up after 3 months of diet and
lifestyle modification plus pharmacotherapy. In addition to a HbA1c of 8.2% at the time, he had normal general tests including
lipid profile, creatinine, urine albumin-to-creatinine ratio, and liver function tests. Which of the following tests is most
appropriate at this time?
Lipid profile
HbA1c
Creatinine
495. Clinical sign that increases the likelihood of having bronchial asthma:
496. Specify the target LDL level for coronary heart disease:
497. What is the most common cause of chronic heart failure at the present time:
arterial hypertension;
cardiomyopathy;
498. A 50-year-old female patient consulted a gastroenterologist with complaints of pain behind the sternum, burning sensation
arising from horizontal position at night, especially when bending the trunk. ECG - sinus rhythm. Heart rate - 66 beats per
minute. Heart electrical axis - vertical. Proton pump inhibitor test was positive. Which of the following diagnoses is the most
likely?
duodenal ulcer
chronic duodenitis
erosive gastritis
499. In patients with hypertension, which of the following lifestyle interventions leads to the most significant drop in systolic
blood pressure?
Healthy diet
Weight loss
Smoking cessation
Alcohol reduction
Regular exercise
501. What is the appropriate hours of sleep in one day for newborns?
8 to 10 hours
21 to 22 hours
15 to 18 hours
5 to 6 hours
10 to 13 hours
502. Which of the following diagnoses in a patient with low back pain does NOT require urgent care?
Epidural abscess
Vertebral osteomyelitis
503. What factor is associated with reducing the risk of developing pneumonia?
Smoking
HIV infection
504. Which one of the listed conditions leads to left to right shunting?
Pulmonary stenosis
505. Grade I hypertension is diagnosed when systolic blood pressure rises to ... mm Hg. Indicate the correct answer:
159 mm Hg
139 mm Hg
169 mm Hg.
179 mm Hg.
129 mm Hg.
506. With a low TSH level (less than the normal range), which T3 and T4 findings are most suggestive of hyperthyroidism?
Elevated T3 and T4
507. Which of the following is the most common cause of shoulder pain in primary care?
Biceps tendonitis
Glenulohumeral arthritis
Adhesive capsulitis
508. In what period of infectious disease does myocarditis develop most often?
In acute period.
509. A 51-year-old man is hospitalized for acute myocardial infarction. He has decreased cardiac output with hypotension
requiring multiple pressor agents. His urine output drops over the next 3 days. His serum urea nitrogen increases to 59 mg/dL,
with creatinine of 2.9 mg/dL. Urinalysis reveals no protein or glucose, a trace blood, and numerous hyaline casts. Five days later,
he develops polyuria and his serum urea nitrogen declines. Which of the following pathologic findings in his kidneys is most
likely to have caused his azotemia?
Patchy tubular necrosis
Hyperplastic arteriolosclerosis
510. Which of the following is correct regarding oxygen saturation measurement in the newborn baby?
Post-ductal saturations should always be higher than pre-ductal saturations due to shunting across the arterial duct
In a premature baby with a symptomatic PDA there is usually a drop in saturations between pre and post ductal measurements
511. Which of the following is a limitation of measuring protein excretion by using a 24-hour urine collection?
512. In patients with type 2 diabetes and albuminuria, which of the following is the most appropriate first-line pharmacotherapy
for hypertension?
Loop diuretics
Beta blockers
Thiazide diuretics
513. Which of the following medications improves cardiovascular outcomes in patients with hypertension who have myocardial
infarction?
Disappears
Amplified
515. Which of the following is NOT an important area that a doctor must screen in an adolescent patient?
Eating disorders
Emotional abuse
Asthma
Hypertension
Physical abuse
516. A 40-year-old female patient complains of chilliness, drowsiness, and constipation. An objective examination revealed an
enlarged thyroid gland and suspected primary hypothyroidism. To confirm this diagnosis the most informative is:
Determination of T3
517. In heart failure, how do the kidneys try to compensate for the heart's decreased ability to pump blood?
Release epinephrine
518. Which of the following is NOT a common laboratory test in the workup of poor weight gain?
Erythrocyte sedimentation rate
Urinalysis
C-reactive protein
Ultrasound
Rupture of the intima of the artery with the formation of a false lumen of the vessel
The appearance of chest pain during exercise and the absence of changes on the ECG
The appearance of chest pain and depression on the ECG of the ST segment by 1 mm or more
The appearance of a pathological Q wave in the III art. and avF leads
Nifedipine
Carvedilol
Amlodipine
Enalapril
Furosemide
522. Which of the given congenital heart diseases mainly causes obstruction to blood flow from the left ventricle?
Truncus arteriosus
Aortic stenosis
Patent ductus arteriosus
523. Which of the following tests is NOT included in the initial evaluation of all patients with newly diagnosed hypertension?
Electrocardiogram
Urinalysis
Echocardiography
Lipid profile
Fasting glucose
524. Which of the following is the most appropriate pharmacotherapy for a diabetic patient with HbA1c of 9.5% and failure of 2
oral anti-DM drugs?
Addition of a sulfonylureas
Addition of a thiazolidinedione
525. Which group of the following drugs is the first line of treatment for arterial hypertension?
Antiplatelet agents
ACE inhibitors
Loop diuretics
If-channel inhibitors
526. Male, 67 years old, rapid breathing, sudden shortness of breath, hypoxemia and diffuse crackles on auscultation.
Treatment measures?
527. What is the name of a primary chronic non-coronary, non-inflammatory, non-metabolic heart disease, manifested by
diffuse myocardial lesion with a sharp decrease in its contractility and progressive dilation of cavities:
restrictive cardiomyopathy
hypertrophic cardiomyopathy
dilated cardiomyopathy
CHD
myocarditis
528. Which of the following statements regarding the specialty of family medicine is most accurate?
The most important aspect of family physician care is the use of technology
529. Which of the following evaluations is recommended by most guidelines for all children under the age of 5 years?
Visual assessment
Lipid screening
Level bleeding
Angina pectoris
Circulatory failure
Thromboembolism
531. The simplest and most accessible method for determining arterial blood oxygen saturation:
peak flowmetry;
pulse oximetry;
spirometry;
pneumotachometry;
thermometry.
534. According to this lecture and based on some guidelines, what is the approximate recommended age to screen for anemia
in children?
5 years
8 years
12 years
3 years
1 year
535. A 50-year-old patient with a bicycle ergometric test with a capacity of 900 kgm/min developed horizontal depression of the
segment ST. To evaluate the results of the test.
536. Which of the following is true regarding protein excretion measurement using the protein-to-creatinine ratio?
537. A patient with late-stage cirrhosis develops portal hypertension. Which of the following options below are complications
that can develop from this condition? Select all that apply:
Ascites
Splenomegaly
Esophageal varices
Hypertrophic cardiomyopathy
Hypertension
Penicillin allergy
Aneurysm